SlideShare uma empresa Scribd logo
1 de 35
Baixar para ler offline
Hình học kì 1 nâng cao Thầy Hồng Trí Quang
1
Nội dung chuyên đề hình học bao gồm
1. Hệ thức lượng trong tam giác vuông
2. Tỉ số lượng giác
3. Đường tròn
a. Định nghĩa và tính chất của đường tròn
b. Tiếp tuyến và phương pháp chứng minh tiếp tuyến
c. Tính chất hai tiếp tuyến cắt nhau
d. Đường tròn nội tiếp tam giác
e. Vị trí tương đối của hai đường tròn
HỆ THỨC LƯỢNG TRONG TAM GIÁC VUÔNG
Cho tam giác ABC vuông tại A, đường cao AH. Chứng minh:
1) AC2
= CH . CB AB2
= BH . BC
2) AH2
= HB . HC
3) AH.BC= AB.AC
4) 2 2 2
1 1 1
AH AB AC
 
5) BC2
= AC2
+ AB2
(Định lý Pi-ta-go)
Bài 1. Cho hình vuông ABCD. Gọi E là một điểm nằm giữa A, B. Tia DE và tia CB cắt nhau ở
F. Kẻ đường thẳng qua D vuông góc với DE, đường thẳng này cắt đường thẳng BC tại G. Chứng
minh rằng:
a) Tam giác DEG cân
b) Tổng 2 2 2
1 1 1
DE DF DC
 
c) Một đường thẳng Ax thay đổi đi qua A sao cho Ax cắt đoạn DC tại M và cắt đường thẳng BC
tại N. Chứng minh rằng tổng 2 2
1 1
AM AN
 là không đổi.
yx
6
4
H CB
A
Hình học kì 1 nâng cao Thầy Hồng Trí Quang
2
a) Ta có: 1 3D D (cùng phụ với 2D )
xét àADE v CDG  ta có :    1 3
0
( )
. .
90
AD DC gt
D D cmt ADE CDG g c g
A C


      

    
DE DG DEG    cân tại D
b) vì DE = DG 2 2
1 1
DE DG
  ta có : 2 2 2 2
1 1 1 1
DE DF DG DF
  
Xét tam giác DGF vuông tại D, ta có : 2 2 2
1 1 1
CD DG DF
  (định lý 4)
c) Từ A kẻ đường thẳng vuông góc với AM, cắt BC tại P, chứng minh tương tự ta có:
2 2 2
1 1 1
AM AN a
  không đổi
Bài 2. Cho hình vuông ABCD. Trên cạnh BC lấy điểm M, trên cạnh CD lấy điểm N. Tia AM
cắt đường thẳng CD tại K. Kẻ AI vuông góc với AK cắt CD tại I. Biết 𝑀𝐴𝑁̂ = 450
1. Chứng minh: IN = MN và MN = ND + BM
2. MN = 5 cm, CM - CN = 1 cm. Tính diện tích tam giác AMN.
3. Từ điểm O trong tam giác AIK kẻ OP, OQ, OR lần lượt vuông góc với IK, AK, AI ( P
IK, QAK, R AI). Xác định vị trí điểm O để 222
OROQOP  nhỏ nhất khi O, M di động.
Tìm giá trị nhỏ nhất đó.
3
2
1
G
F
E
D C
BA
Hình học kì 1 nâng cao Thầy Hồng Trí Quang
3
Ta có
)1(AIAMADIABM  và ID = BM
Ta có MN = IN = ID + DN = BM + DN
b) Do 2 2
25CM CN  và CM - CN = 1 CM = 4; CN = 3 MN = 5
Từ đó tính diện tích AMN qua phần bù.
)(155.6.
2
1
.
2
1 2
cmMNAHS AMN 
3. Từ giả thiết ta có AQOR là hình chữ nhật
222
)( 222
22222 ADAPOPOA
OPOAOROQOP 


222
OROQOP  nhỏ nhất khi O là trung điểm của AD.
Bài 3. Cho tam giác ABC vuông tại A trung tuyến AM.
a) Chứng minh rằng
1
2
AM BC
b) 0
15ABC  . Chứng minh rằng: 2
4 .BC AB AC
HD Kẻ đường trung tuyến AM, đường cao AH. Tính
1 1
2 4
AH AM BC  . Sử dụng diện tích tam
giác ABC.
c) Nếu tam giác ABC vuông tại A có trung tuyến BM, 0
15ABM  và 16ABCS  . Tính độ dài
BM
d) Nếu 0
75BAC  , đường cao CH thỏa mãn:
1
.
2
CH AB Chứng minh tam giác ABC cân.
A B
CD
KI
M
H
N
Hình học kì 1 nâng cao Thầy Hồng Trí Quang
4
b) 21
8
8
ABCS BM 
d) Kẻ BD vuông AC, DP vuông AB thì DP là đường trung bình trong tam giác ABD.
Tự luyện
Bài 4. Cho tam giác ABC có AB = 6cm, AC = 8cm, các đường trung tuyến BD và CE vuông
góc với nhau.
a) Tính độ dài BC. Đs 𝐵𝐶 = 2√5
b) Tính độ dài AB, AC
Bài 5. Tính cạnh đáy BC của tam giác cân ABC biết đường cao tương ứng với cạnh đáy bằng
15,6cm và đường cao tương ứng với cạnh bên bằng 12cm. Đs BC = 13cm
Bài 6. Cho M thuộc miền trong của hình chữ nhật ABCD. Chứng minh rằng: 𝑀𝐴2
+ 𝑀𝐶2
=
𝑀𝐵2
+ 𝑀𝐷2
HD kẻ qua M đt song song với AB cắt AD, BC tại E và F. Sử dụng Pitago
Bài 7. Cho hai điểm A, B cố định và điểm M di động sao cho tam giác MAB có ba góc nhọn.
Gọi H là trực tâm của tam giác MAB và K là chân đường cao vẽ từ M của tam giác MAB. Tìm
GTLN của tích KH.KM./.
H
K
A B
M
Hình học kì 1 nâng cao Thầy Hồng Trí Quang
5
Ta có: BKM   HKA (g.g)
 . .
BK KM
BK KA KM KH
HK KA
  
Mặt khác: BK.KA
2 2
2 4
BK KA AB 
  
 
. Dấu “=” xảy ra khi BK = KA
2
.
4
AB
KM KH  .
Vậy max (KM.KH) =
2
4
AB
khi BK = KA, tức là K là trung điểm của AB.
Bài 8. Cho tam giác ABC, gọi I là tâm đường tròn nội tiếp tam giác. Qua I dựng đường thẳng
vuông góc với IA cắt AB, AC tại M và N. Chứng minh rằng :
a)
2
2
BM BI
CN CI
 b) BM.AC + CN.AB + AI2
= AB.AC
a) I là tâm đường tròn nội tiếp  ABC  AI là phân giác A ; MN AI
0 1 1
90 ( )
2 2
AMI A B C    
1 1
2 2
AMI MIB B MIB C    
BMI BIC    2
.
BM BI
BI BM BC
BI BC
   (1)
Tương tự 2
.CI CN CB (2)
Từ (1) và (2) suy ra
2
2
BM BI
CN CI

b) Từ chứng minh trên BMI INC  
B C
A
IM
N
Hình học kì 1 nâng cao Thầy Hồng Trí Quang
6
. .
BM MI
BM CN MI NI
IN NC
  
Mà AMN cân IM = IN BM.CN = IM2
Xột  AIM có: AI2
+ IM2
= AM2
2 2 2
2 2 2
. .
IM AM AI
BM CN AM AI AM AN AI
  
    
=(AB-BM)(AC- CN) – AI2
= AB.AC-AB.CN- BM.AC+BM.CN-AI2
BM.AC +CN.AB +AI2
= AB.AC (đpcm)
TỈ SỐ LƯỢNG GIÁC CỦA GÓC NHỌN
Cho tam giác ABC vuông tại A. Ta có tỉ số lượng giác của góc nhọn:
sin
AC
B
BC
 cos
AB
B
BC
 tan
AC
B
AB
 cot
AB
B
AC

Công thức cơ bản:
2 2
sin cos 1B B  tan .cot 1B B 
sin cosB C và sin cosC B
Công thức nâng cao
2
2
1
1 tan
cos x
  2
2
1
1 cot
sin x
 
Diện tích tam giác (trong đó r là bán kính đtròn nội tiếp)
1 1
. . .sin .
2 2
S a h b c A p r  
Bán kính ngoại tiếp R 2
sin sin sin
a b c
R
A B C
   sin
2
a
A
R

Bài 9. Cho tam giác ABC có 3 góc nhọn, vẽ đường cao AD và BE. Gọi H là trực tâm của tam
giác ABC. Gọi a, b, c lần lượt là độ dài các cạnh BC, CA, AB của tam giác ABC
a) Chứng minh: tanB.tanC =
AD
HD
b) Chứng minh:
2
.
4
BC
DH DA 
Hình học kì 1 nâng cao Thầy Hồng Trí Quang
7
Ta có tanB =
AD
BD
; tanC =
AD
DC
 tanB.tanC =
2
.
AD
BD DC
(1)
Xét 2 tam giác vuông ADC và BDH có DAC DBH vì cùng phụ với góc C nên ta có :
AD BD
ADC BDH
DC DH
    . .AD DH DB DC  
2
.
AD AD
BD DC HD
 (2)
Từ (1) và (2)  tanB.tanC =
AD
HD
.
Theo câu a. ta có:
2 2
( )
. .
4 4
DB DC BC
DH DA DB DC

  
Bài 10. Cho tam giác ABC có 3 góc nhọn. Chứng minh rằng:
bc
aA
22
sin 
K
G
H
E
D
A
B C
x
F
M
N
A
B
C
Hình học kì 1 nâng cao Thầy Hồng Trí Quang
8
Gọi Ax là tia phân giác góc A, kẻ BM; CN lần lượt vuông góc với Ax
Ta có sin sin
2
A BM
MAB
AB
  suy ra .sin
2
A
BM c
Tương tự .sin
2
A
CN b do đó ( ).sin
2
A
BM CN b c  
Mặt khác ta luôn có: BM CN BF FC BC a    
Nên ( ).sin
2
A
b c a  sin
2 2 .
A a a
b c b c
  

Bài 11. Tính: sin2
150
+ sin2
250
+ sin2
350
+ sin2
450
+ sin2
550
+ sin2
650
+ sin2
750
= sin2
150
+ sin2
250
+ sin2
350
+ sin2
450
+ sin2
550
+ sin2
650
+ sin2
750
= (cos2
750
+ sin2
750
) + (cos2
650
+ sin2
650
) +(cos2
550
+ sin2
550
) + sin2
450
= 1 + 1 + 1 +
1
2
=
1
3
2
Bài 12. Cho tam giác ABC
a) Tính 0
sin15
b) Nếu tam giác ABC vuông tại A thỏa mãn 2
4 .BC AB AC thì tam giác ABC có một góc bằng
0
15
c) Nếu 0
45BAC  , 0
75ABC  . Trên cạnh AB lấy điểm M sao cho
1
.
3
AM AB Tính ACM
a) Tính tỉ số lượng giác,
c) Kẻ MN, BD vuông AC, đặt AN = x. Tính MC, MN, NC theo x có: 2
4 .MC MN NC
Tự luyện
Bài 13. Cho hình vuông ABCD, biết M, N theo thứ tự là trung điểm BC, CD.
a) Tính độ dài AM, AN, MN.
b) Gọi K là giao điểm của AC và MN, tính AK
Hình học kì 1 nâng cao Thầy Hồng Trí Quang
9
c) Gọi H là hình chiếu của M trên AN, tính MH.
d) Tính cos 𝑀𝐴𝑁̂
Bài 14.
a) *Cho biết sinx = 0,6. Tính cosx, tanx và cotx
b) Tính 2 0 2 0 2 0 2 0 2 0 2 0 2 0
15 25 35 45 55cos cos cos cos c 65 7s co 5o ssin     
+ ta có: 2 2 2 2
sin cos 1 cos 1 sin 1 0,6 0,8          
+
sin 0,6 3 cos 0,8 4
tan ;
cos 0,8 4 sin 0,6 3
cot
 
 
 
     
Bài 15. Góc nhọn của một hình thang cân bằng 600
, đường phân giác của góc nhọn này chia
đường chéo của hình thang cân theo tỉ số 4:11 và chia đáy thành hai đoạn mà hiệu độ dài hai
đoạn này bằng 6cm.
a) Chứng minh rằng DC = AB + AD; b) Tính các cạnh đáy của hình thang
HD b) DC = 66cm; AB = 42cm.
Bài 16. Cho tam giác ABC vuông tại A có trọng tâm G; BD là phân giác góc B, GD vuông góc
với AC.
a) Gọi E là trung điểm AG, chứng minh rằng ED song song BC
b) Tính các góc của tam giác ABC.
HD b) chứng minh góc ACB = 300
Bài 17. Cho tam giác ABC vuông tại A có đường cao AH. Gọi D là điểm đối xứng với A qua B.
Gọi E là điểm thuộc tia đối của tia HA sao cho HE = 2HA và I là trung điểm HE. Chứng minh
rằng
a) tan 𝐼𝐸𝐷̂ = tan 𝐻𝐶𝐸̂; b) Tam giác DEC vuông.
HD AH = a, HB = b; tan = tan = 2b/a.
Bài 18. Góc nhọn của một hình thang cân bằng 600
, đường phân giác của góc nhọn này chia
đường chéo của hình thang cân theo tỉ số 4:11 và chia đáy thành hai đoạn mà hiệu độ dài hai
đoạn này bằng 6cm.
a) Chứng minh rằng DC = AB + AD; b) Tính các cạnh đáy của hình thang
HD b) DC = 66cm; AB = 42cm.
Hình học kì 1 nâng cao Thầy Hồng Trí Quang
10
Bài 19. Cho tam giác ABC vuông tại A có trọng tâm G; BD là phân giác góc B, GD vuông góc
với AC.
c) Gọi E là trung điểm AG, chứng minh rằng ED song song BC
d) Tính các góc của tam giác ABC.
HD b) chứng minh góc ACB = 300
Bài 20. Cho tam giác ABC vuông tại A có đường cao AH. Gọi D là điểm đối xứng với A qua B.
Gọi E là điểm thuộc tia đối của tia HA sao cho HE = 2HA và I là trung điểm HE. Chứng minh
rằng
a) tan 𝐼𝐸𝐷̂ = tan 𝐻𝐶𝐸̂; b) Tam giác DEC vuông.
HD AH = a, HB = b; tan = tan = 2b/a.
Bài 21. Tính 0 0
sin22 25';tan22 25' mà không dùng bảng số, không dùng máy tính?
Bài 22. Cho tam giác ABC vuông tại A có đường cao AH và 2
4 .AH AM AN , trong đó M, N
theo thứ tự là chân đường vuông góc hạ từ H đến AC, AB. Tính số đo các góc tam giác ABC.
HD góc 150
; 750
Bài 23. Cho tam giác ABC vuông tại A có BE là phân giác trong, I là tâm đường tròn nội tiếp.
Biết
3 1
3 1
BI
EI



. Tính số đo góc ACB?
HD đặt tỉ số
BI
x
EI
 , tính được 2
4 .BE AB AE nên 0
60ACB 
Luyện tập
Bài 24. Ứng dụng hệ thức 0
tan15
Hình học kì 1 nâng cao Thầy Hồng Trí Quang
11
B
E
H
F
C
A
a) Chứng minh rằng:

0 6 4
sin15
2
b) Tính 0
tan15
c) Cho tam giác ABC cân tại B,  0
30BAC . Trên cạnh BC lấy điểm D sao cho 
2
2
AC
BD .
Tính số đo góc CAD?
Hướng dẫn:
a) Dựng tam giác ABC vuông tại A, có  0
30B . Dựng phân giác BD, khi đó:
 0
sin15 sinABD
AD
BD
b) Đs

  

0 6 4
tan15 2 3
6 4
c)
Kẻ AH vuông BC, đặt AH = x. Tính được DH = x nên  0
60DAC
Bài 25. Cho tam giác ABC vuông ở A, AH  BC, HE  AB, HF  AC ( H  BC,
E  AB, F  AC).
a. Chứng minh rằng: AE.AB = AF.AC; BH = BC.cos2
B.
b. Chứng minh rằng:
3
3
AB BE
CFAC
 .
c. Chứng minh rằng:
33 32 2 2
BC CF BE  .
d. Cho BC = 2a. Tìm giá trị lớn nhất của diện tích tứ giác AEHF.
a. AHB vuông tại H, có HE  AB nên AH2
= AB.AE. (1)
Tương tự: AH2
= AC.AF (2).
Từ (1) và (2) suy ra: AB.AE = AC.AF.
Hình học kì 1 nâng cao Thầy Hồng Trí Quang
12
* BH = AB.cosB; AB = BC.cosB
Suy ra BH = BC.cos2
B.
b.(1,5đ).
Áp dụng hệ thức lượng trong tam giác vuông:
AB2
= BC.BH; AC2
= BC.CH;
BH2
= AB.BE; CH2
= AC.CF
nên
2 4 2 3
2 4 2 3
.
.
.
AB BH AB BH AB BE AB BE
CH AC CF CFAC AC CH AC
     
a. (1,5đ)
Ta có BE = BH.cosB; BH = AB.cosB; AB = BC.cosB;
Do đó: BE = AB.cos2
B = BC.cos3
B  BE2
= BC2
.cos6
B

3 32 2 2
.cos .BE BC B
Tương tự ta có:
3 32 2 2
.sin .CF BC B

3 3 3 32 2 2 2 2 2
.(cos sin .)BE CF BC B B BC   
b. (1,0đ)
Ta có: SAEHF = AE.AF
Lại có:
2
AH
AE
AB
 Tương tự:
2
AH
AF
AC

Do đó:
4 4 3 3 3 2
.
. . 2 2
AEHF
AH AH AH AO a a
S
AB AC BC AH BC BC a
     
Max SAEHF =
2
2
a
 ABC vuông cân tại A.
Bài 26. *Cho tam giác ABC cân tại A có 0ˆA 20 ;AB AC b;BC a    . Chứng minh rằng: a3
+ b3
= 3ab2
Hình học kì 1 nâng cao Thầy Hồng Trí Quang
13
 ABC cân tại A có góc BAC = 200
nên ABC = ACB = 800
Trên cạnh AC lấy D sao cho ABD = 600
, khi đó DBC = 200
nên
BDC = 800
  BDC cân tại B  BD = BC = a .
 BDC  ABC ( g – g) 
DC BC
BC AC
  DC =
2
a
b
 AD = b -
2
a
b
 BDE vuông có 0
60EBD  nên BE =
1
2
BD =
1
2
a và DE = BD
3
2
= a.
3
2
; AE = b -
1
2
a.
Áp dung định lý Pi-ta-go trong tg vuông ADE có :
AD2
= AE2
+ DE2
 (b -
2
a
b
)2
= (b -
1
2
a)2
+ (a.
3
2
)2
 b2
- 2a2
+
4
2
a
b
= b2
- ab +
2
4
a
+
2
3
4
a

4
2
a
b
= 3a2
–ab
 a4
= 3a2
b2
- ab3
 a4
+ ab3
= 3a2
b2
 a3
+ b3
= 3ab2
Tự luyện
Bài 27. Tính cos 36, cos 72 mà không dùng bảng số, không dùng máy tính?
HD vẽ tam giác cân ABC tại A có 𝐴 = 360
, phân giác CD, đường cao DH, AD = DC = CB = 1;
0
os36
D
AH
c
A

0 1 5
os36
4
c

0 5 1
os72
4
c
Bài 28. Cho tam giác ABC có ba góc nhọn và có độ dài 3 cạnh BC, AC, AB lần lượt bằng a, b,
c. Chứng minh rằng:
))((.. SinCSinBSinAcbaSinCcSinBbSinAa 
Vẽ đường cao AH. Ta có
SinC
c
SinB
b
HC
AH
SinC
HB
AH
BSin  ...;
Tương tự, suy ra: 0


 k
SinCSinBSinA
cba
SinC
c
SinB
b
SinA
a
Vậy kSinCSinBSinAcSinCbSinBaSinA ).(...  (1)
Và (a + b + c) = (SinA + Sin B + SinC).k
B C
A
D
E
Hình học kì 1 nâng cao Thầy Hồng Trí Quang
14
Suy ra: kSinCSinBSinASinCSinBSinAcba ).())((  (2)
Từ (1) và (2) ta có đpcm
Bài 29. Cho tam giác ABC nhọn, đường cao AH. Điểm M thuộc đoạn thẳng BC. Kẻ MK vuông
góc AB, ML vuông góc AC ( K thuộc AB, L thuộc AC ). Đường thẳng qua A và vuông góc với
AM cắt MK, ML thứ tự tại E, F. Từ B kẻ đường thẳng vuông góc với CE, cắt Ah tại I. Chứng
minh rằng:
a) Tam giác AIB đồng dạng với tam giác MCE
b)
EM ML BM AI
và
FM KM FM AC
 
c) Ba đường thẳng AH, BF, CE đồng quy.
Bài 30. Cho tam giác ABC có đường phân giác AD, đường cao BH, đường trung tuyến CE đồng
quy tại O. Vẽ EF vuông góc với BH (FBH). Chứng minh:
a) CH.AE = EF.AC
b) . os .cos . osAC c BAC BD ACB CDc ACB 
Bài 31. Cho tam giác ABC có 0
15CAB  ; 0
30ABC  . Gọi M là trung điểm cạnh AB
a) Tính số đo góc ACM
b) Chứng minh rằng:
.
2
AB BC
CM
AC

HD kẻ MN, BD vuông góc AC.Đặt CD = BD = x, đs 0
30MCA 
Bài 32. Trên cạnh BC của tam giác ABC lấy điểm P sao cho PC = 2PB. Tính số đo góc ACB nếu
0
45ABC  , 0
60CPA 
HD Kẻ PQ, CH vuông AB, đặt QB = QP = x. Đs 0
75BCA 
ĐƯỜNG TRÒN
Bài 1. Định nghĩa tính đối xứng của đường tròn
Bài 33. Cho tam giác ABC nhọn, vẽ đường tròn tâm O đường kính BC cắt các cạnh AB, AC theo
thứ tự tại D và E
a) Chứng minh rằng CD vuông góc với AB; BE vuông góc với AC
b) Gọi K là giao điểm của BE và CD. Chứng minh rằng: AK vuông góc với BC
LG
Hình học kì 1 nâng cao Thầy Hồng Trí Quang
15
a) Theotính chất tam giác BCD và tam giác BCE có cạnh BC là đường kính => tam giác BCD
vuông tại D (=> CD vuông góc với AB) và tam giác BCE vuông tại E (=> BE vuông góc với AC)
b) Xét tam giác ABC, ta có :
à
BE AC
CD AB
m BE CD K
 

 
  
K là trực tâm của tam giác ABC => AK vuông góc với BC
Bài 34. *Cho tam giác cân ABC (AB = AC). Gọi H là trực tâm tam giác, gọi D, M, N lần lượt là
trung điểm của BC, AB, CH. Chứng minh rằng trung điểm của MN là tâm đường tròn ngoại tiếp
tam giác DMN.
Bài 35. *Cho tam giác nhọn ABC nội tiếp đường tròn (O; R). Gọi M là trung điểm BC. Giả sử O
nằm trong tam giác AMC hoặc O nằm giữa A và M. Gọi I là trung điểm AC. Chứng minh rằng:
a) Chứng minh MA + MC > OA + OC
b) Chu vi tam giác IMC lớn hơn 2R
c) Chu vi tam giác ABC lớn hơn 4R
HD IM + IC + MC = IM + IA + IC > MA + MC > OA + OC = 2R
Bài 36. *Cho hai đường thẳng xy và x’y’ vuông góc nhau tại O. Một đoạn thẳng AB=6cm
chuyển động sao cho A luôn nằm trên xy và B trên x’y’. Hỏi trung điểm M của AB chuyển động
trên đường nào?
Bài 37. *Cho ba điểm A, B, C bất kì và đường tròn (O) bán kính 1. Chứng minh rằng tồn tại
điểm M nằm trên đường tròn (O) sao cho: MA + MB + MC ≥ 3.
Hd. Sử dụng nguyên lí Drichle, gọi đường kính DE và chỉ ra
(𝐷𝐴 + 𝐷𝐵 + 𝐷𝐶) + (𝐸𝐴 + 𝐸𝐵 + 𝐸𝐶) ≥ 3
Bài 2. Tính chât đối xứng của đường tròn
K
E
D
O
CB
A
Hình học kì 1 nâng cao Thầy Hồng Trí Quang
16
1. Đường kính vuông góc dây cung thì đi qua trung điểm và ngược lại.
2. Hai dây cung bằng nhau khi và chỉ khi chúng cách đều tâm.
3. Dây cung nào gần tâm hơn thì dài hơn và ngược lại.
4. Số đo của cung bằng số đo góc ở tâm
5. Hai đường thẳng song song cắt đường tròn tạo thành hai cung có số đo bằng nhau
Bài 38. Cho đường tròn tâm O đường kính AB = 13cm. Dây CD có độ dài 12cm vuông góc với
AB tại H.
a) Tính các độ dài đoạn HA, HB
b) *Gọi M, N theo thứ tự là hình chiếu của H trên AC, BC. Tính diện tích tứ giác CMHN.
HD b) ABCS 39 ;
2 2
CHN
ABC
S CH 6
S AH 13
   
    
   
CHN ABC
36 108
S .S
169 13
   ; 2
CMHN
8
S 16 cm
13

Bài 39. Cho tam giác ABC nội tiếp đường tròn (O) có D, E, F theo thứ tự là trung điểm của BC,
AC, AB.
a) Gọi H là trực tâm của tam giác. Chứng minh hai tam giác HAB và ODE đồng dạng
b) Chứng minh
1
OD / /AH,OD AH
2

c) Kẻ các đường thẳng DD’, EE’, FF’ sao cho DD’//OA, EE’//OB, FF’//OC. Chứng minh rằng
các đường DD’, EE’, FF’ đồng quy.
Bài 40. Cho nửa đường tròn tâm O đường kính AB, dây CD. Gọi H, K là chân các đường vuông
góc kẻ từ A, B đến CD. Gọi I là trung điểm CD
a) Chứng minh rằng IH = IK
b) Chứng minh rằng CH = DK
b) *Chứng minh rằng: AHKB ACB ADBS S S 
c) Bết AB = 30cm, CD = 18cm, tính OI và diện tích lớn nhất của tứ giác AHKB.
Hd a) gọi I là trung điểm CD b) Qua I kẻ các đường thẳng song song với AB, khi đó
AHKB AEFBS S
c) ta có IO = 12cm, AHKBS AB.II' AB.IO 30.12 360   
Hình học kì 1 nâng cao Thầy Hồng Trí Quang
17
Bài 41. Cho tam giác cân ABC (AB = AC) nội tiếp đường tròn (O). Gọi M là trung điểm của
cạnh AB, G là trọng tâm tam giác AMC.
a) Chứng minh OA vuông góc với MG
b) Chứng minh EG song song với AB
c) Chứng minh 𝑂𝐺 ⊥ 𝑀𝐶.
Bài 42. Cho tam giác ABC, các đường cao AD, BE, CF. Đường tròn tâm (O) đi qua D, E, F cắt
BC, CA, AB theo thứ tự tại M, N, P. Kẻ MM’, NN’, PP’ lần lượt vuông góc với BC, CA, AB.
a) Gọi I là giao điểm của MM’ và NN’, chứng minh O là trung điểm HI
b) Chứng minh ba đường MM’, NN’, PP’ đồng quy
Hd; Gọi O’ là trung điểm của HI suy ra O thuộc trung trực của dây DM, O cũng thuộc trung
trực của dây EN. Do đó O’ cũng là tâm đường tròn đi qua sáu điểm D, M, N, E, F, P; và điểm I
là điểm đối xứng với H qua O
Chứng minh tương tự MM’ cắt PP’ tại I’ thì I’ cũng đxứng với H qua O. Do đó I trùng I’
Vậy 3 đt MM’, NN’, PP’ đồng quy tại I
Vấn đề 3: vị trí tương đối giữa đường thẳng d và đường tròn. (O; R)
1. Cho đường tròn (O; R) và đường thẳng d khi đó có các trường hợp sau:
1.1. Nếu d(O;Δ) = OH > R thì đường thẳng và đường tròn không có điểm chung. Ta nói đường
thẳng và đường tròn ngoài nhau hoặc không cắt nhau.
1.2. Nếu d(O; Δ) = OH = R khi đó đường thẳng và đường tròn có một điểm chung duy nhất
chính là H. Khi đó ta nói đườngthẳng tiếp xúc đường tròn (đường thẳng này gọi là tiếp tuyến
của (O)).
1.3.Nếu d(O; Δ) = OH < R thì đường thẳng d cắt đường tròn (O; R) tại hai điểm phân biệt A và
B. Đường thẳng này gọi là cát tuyến với (O; R).
2. Vậy muốn xác định vị trí của đường thẳng d và đường tròn ta cần tìm bán kính R và khoảng
cách d(O; d) rồi so sánh và kết luận.
Bài 43. Cho Δcân OAB có OA=OB=5cm và AB=6cm. Hỏi bán kính R của đường tròn (O; R) có
giá trị bao nhiêu để đường tròn tiếp xúc AB.
Bài 44. Cho nửa đường tròn tâm O bán kính R, đường kính AB, M là một điểm thuộc nửa đường
tròn. Qua M vẽ tiếp tuyến với nửa đường tròn. Gọi D và C theo thứ tự là các hình chiếu của A và
B trên tiếp tuyến ấy.
a) Chứng minh M là trung điểm CD
Hình học kì 1 nâng cao Thầy Hồng Trí Quang
18
b) Chứng minh AB = BC + AD
c) Giả sử AOM BOM , gọi E là giao điểm của AD với nửa đường tròn. Xác định dạng của tứ
giác BCDE.
d) Xác định vị trí điểm M trên nửa đường tròn ấy sao cho tứ giác ABCD có diện tích lớn nhất.
Tính diện tích đó theo R
Bài 45. Cho nửa đường tròn tâm O đường kính AB, Ax là tiếp tuyến của nửa đường tròn, C là
một điểm thuộc nửa đường tròn, H là hình chiếu của C trên AB. Đường thẳng qua O vuông góc
với AC cắt Ax tại M. Gọi I là giao điểm của MB và CH.
a) Gọi N là giao điểm của BC và Ax, chứng minh MA = MN.
b) Chứng minh I là trung điểm của CH.
Tự luyện
Bài 46. Cho đường tròn tâm O có đường kính AB = 2R. Gọi d là tiếp tuyến của đường tròn, A là
tiếp điểm. Gọi M là điểm bất kì thuộc d. Qua O kẻ đường thẳng vuông góc với BC, cắt d tại N.
a) Chứng minh tích AM.AN không đổi khi M chuyển động trên d
b) Tìm giá trị nhỏ nhất của MN
Bài 47. Cho nửa đường tròn tâm O đường kính AB. Vẽ các tiếp tuyến Ax, By với nửa đường
tròn. Trên nửa đường tròn lấy điểm C. Các tia BC và AC lần lượt cắt Ax, By tại D và E. Gọi M,
N theo thứ tự là trung điểm của AD và BE.
a) Chứng minh rằng MN là tiếp tuyến của đường tròn (O)
b) Vẽ CH vuông góc với AB, chứng minh ba đường CH, AN, BM đồng quy
HD a) Chứng minh M, C, N thẳng hàng (tổng hai góc vuông); b) Chứng minh cùng đi qua trung
điểm K của CH.
Bài toán Chứng minh đường thẳng d là tiếp tuyến(O; R)
Cách 1. Xác định H thuộc d sao cho OH d và chứng minh OH = R
Cách 2. Xác định H thuộc d sao cho OH = R và chứng minh OH d
Bài 48. Cho hình thang vuông ABCD tại A và D, có 0
90BMC với M là trung điểm AD.
a) Cách 2. Chứng minh rằng AD là tiếp tuyến của đường tròn đường kính BC
b) Cách 1. Chứng minh rằng BC là tiếp tuyến của đường tròn đường kính AD.
Hình học kì 1 nâng cao Thầy Hồng Trí Quang
19
Bài 49. Cho nửa đường tròn tâm O đường kính AB, C là một điểm thuộc nửa đường tròn, H là
hình chiếu của C trên AB. Qua trung điểm M của CH kẻ đường vuông góc với OC, cắt nửa
đường tròn tại D và E.
a) Chứng minh rằng CO.CK = CM.CH
b) OC cắt DE ở K và cắt (O) tại I. Chứng minh rằng 2
2 .CE COCK
c) Chứng minh AB là tiếp tuyến của đường tròn tâm C bán kính CD. Kí hiệu (C;CD)
Bài 50. Cho tam giác ABC vuông tại A (AB < AC), đường cao AH. Gọi E là điểm đối xứng với
B qua H. Đường tròn có đường kính EC cắt AC tại K. Chứng minh rằng HK là tiếp tuyến của
đường tròn
HD Gọi O là trung điểm EC, M là trung điểm AK. Chứng minh 0
90OKH
Hai Tiếp tuyến cắt nhau
Bài 51. Cho nửa đường tròn (O ; R) đường kính AB. Gọi Ax, By là các tia vuông góc với AB
(Ax, By và nửa đường tròn cùng thuộc nửa mp có bờ là AB). Lấy M thuộc Ax, qua M kẻ tiếp
tuyến với nửa đường tròn, cắt By tại N
a) Tính góc MON
b) CMR: MN = AM + BN
c) CMR tích AM.BN không đổi
d) Tứ giác AMNB có diện tích nhỏ nhất khi nào? Tính diện tích đó
LG
a) - theo tc của 2 tiếp tuyến cắt nhau, ta có:
1 2
3 4
1
;
2
1
;
2
O O AOH MA MH
O O BOH NB NH
  
  
(1)
- ta có:
  0 0
2 3
1 1
.180 90
2 2
MON O O AOH BOH     
b) do MN = MH + NH (2)
=> từ (1) và (2) : MN = MA + NB
4
32
1
y
x
H
N
M
R BA O
Hình học kì 1 nâng cao Thầy Hồng Trí Quang
20
c) Xét tam giác MON vuông tại O, theo hệ thức về cạnh
và đường cao trong tam giác vuông, ta có :
2
2. .
.
à
OH MH NH AM BN
AM BN R
m OH R
 
 
 
Bài 52. Cho hình vuông ABCD có cạnh bằng a. Với tâm B và bán kính a, vẽ cung AC nằm trong
hình vuông. Qua điểm E thuộc cung đó, vẽ tiếp tuyến với cung AC, cắt DA và DC theo thứ tự
tại M và N.
a) Tính chu vi tam giác DMN
b) Tính số đo góc MBN
c) Chứng minh rằng:
2a
MN a
3
 
Hd Chu vi = 2a, b) góc 450
, c) MN + MN < DM + DN + MN = 2a; MN + MN + MN > DM +
DN + MN
Bài tập nâng cao
Bài 53. Cho đường tròn (O), điểm K bên ngoài đường tròn. Kẻ các tiếp tuyến KA, KB với đường
tròn (A, B là các tiếp điểm). Kẻ đường kính AOC. Tiếp tuyến của đường tròn (O) tại C cắt AB ở
E. Chứng minh rằng:
a) Tam giác KBC đồng dạng với tam giác OBE
b) CK vuông góc với OE
vd 15 – 103. Từ
Toán phát triển 73 – 79 (105)
Bài 54. Cho đoạn thẳng AB. Vẽ về một phía của AB các tia Ax, By song song với nhau
a) Dựng đường tròn (O) tiếp xúc với đoạn thẳng AB và tiếp xúc với các tia Ax, By
b) Tính góc AOB
c) Gọi các tiếp điểm của (O) với Ax, By, AB theo thứ tự là M, N, H. Chứng minh rằng MN là
tiếp tuyến của đường tròn đường kính AB.
d) Các tia Ax, By có vị trí như thế nào để HM = HN.
Hình học kì 1 nâng cao Thầy Hồng Trí Quang
21
Bài 55. Cho hình thang vuông ABCD (vuông tại A, D), tia phân giác góc C đi qua trung điểm I
của AD.
a) Chứng minh rằng BC là tiếp tuyến của đường tròn (I; IA).
b) Cho AD = 2a. Tính tích của AB.CD theo a
c) Gọi H là tiếp điểm của BC với (I) nói trên. K là giao điểm của AC và BD. Chứng minh rằng
HK song song với DC.
Bài 56. Cho đường tròn tâm (O) đường kính AB, D là một điểm nằm trên đường tròn. Các tiếp
tuyến của đường tròn tại A, D cắt nhau ở C. Gọi E là hình chiếu của D trên AB, gọi I là giao
điểm của BC và DE. Chứng minh rằng DI = IE.
Bài 57. Cho tam giác ABC cân tại A, O là trung điểm của BC. Vẽ đường tròn tâm O tiếp xúc với
AB, AC tại H, K. Một tiếp tuyến với đường tròn (O) cắt các cạnh AB, AC ở M, N.
a) Cho B C   . Tính MON
b) Chứng minh rằng OM, ON chia tứ giác BMNC thành ba tam giác đồng dạng.
c) Cho BC = 2a. Tính tích BM.CN
d) Tiếp tuyến MN ở vị trí nào để tổng BM + CN nhỏ nhất
Bài 58. Cho tam giác ABC vuông tại A, đường cao AH, HB = 20cm, HC = 45cm. Vẽ đường tròn
tâm A bán kính AH. Kẻ các tiếp tuyến BM, Cn với đường tròn (M, N là tiếp điểm, khác H).
a) Tính diện tích tứ giác BMNC
b) Gọi K là giao điểm của CN và HA. Tính độ dài AK, KN.
c) Gọi I là giao điểm của AM và CB. Tính độ dài IM.
Bài 59. Cho đường tròn tâm O bán kính 6cm. Một điểm A nằm bên ngoài đường tròn sao cho
các tiếp tuyến AB, AC với đường tròn vuông góc với nhau (B, C là các tiếp điểm). Trên hai
cạnh AB, AC lấy các điểm D, E sao cho AD = 4cm, AE = 3cm. Chứng minh rằng DE là tiếp
tuyến của đương tròn (O).
Ôn tập
Bài 60. Cho đường tròn (O) đường kính AB và tia tiếp tuyến Ax. Từ M trên Ax kẻ tiếp tuyến thứ
hai MC với đường tròn. Đường vuông góc với AB tại O cắt BC ở N.
a) Có nhận xét gì về tứ giác OMNB?
b) Tìm tập hợp điểm H là trực tâm tam giác MAC khi M di động trên Ax.
HD a) H.b.b do MO song song và bằng BN
Hình học kì 1 nâng cao Thầy Hồng Trí Quang
22
b) Tứ giác AOCH là b.b.h nên AH = OC = R. Quỹ tích là (A; R).
Bài 61. Đề thi chọn hsg toán 9 quận Tân Bình 2003 - 2004
Cho tam giác ABC nội tiếp đường tròn tâm O đường kính AC. Trên tia AB lấy điểm D sao cho
AD = 3AB. Đường thẳng Dy vuông góc với DC tại D cắt tiếp tuyến Ax của đường tròn (O) tại
E. Chứng minh tam giác BDE là tam giác cân.
Cách 1. Gọi F là giao điểm CD với (O), H là giao điểm AF và BC, I là trung điểm BD
Chứng minh AHDE là hình bình hành, suy ra EIKB cũng là h.b.b, từ đó EI vuông BD
Cách 2. Hạ EI vuông BD, vẽ JM vuông AD thì M là trung điểm AD
Hình học kì 1 nâng cao Thầy Hồng Trí Quang
23
Cách 3. Hạ EI vuông góc BD, tam giác IDE đồng dạng BCD (g.g). Suy ra
ID IE
BC BD

Tương tự
IA IE
BC AB

Vậy
1
2
ID ID BC IE AB
IA BC IA BD IE
  
Bài 62. Cho đường tròn (O) đường kính AB và tia tiếp tuyến Ax. Từ M thuộc Ax kẻ tiếp tuyến
thứ hai MC với đường tròn. Đường vuông góc với AB tại O cắt BC ở N.
a) Có nhận xét gì về tứ giác OMNB
b) Tìm tập hợp trực tâm H của tam giác MAC khi M di động trên Ax
HD a) h.b.h. b) Cm AOCH là h.b.h mà có OA = OC nên H ở nửa đường tròn tâm A bán kính R
nằm trong nửa mp bờ AB chứa Ax mà trừ điểm O.
Tự luyện
Bài 63. Cho hình thang ABCD vuông tại A và D, tia phân giác của góc C đi qua trung điểm I của
AD.
a) Chứng minh rằng BC là tiếp tuyến của đường tròn (I; IA)
b) Cho AD = 2a. Tính tích của AB và CD theo a
c) Gọi H là tiếp điểm của BC và đường tròn (I) nói trên. K là giao điểm của AC và BD. Chứng
minh rằng HK song song với DC
Hình học kì 1 nâng cao Thầy Hồng Trí Quang
24
Hd a) Kẻ IH vuông BC, chứng minh IH = R
b) Chứng minh góc BIC vuong có AB.CD = HB.HC = IH2
c) Theo Talet
BH BA BK
HC DC KC
 
Bài 64. Cho tam giác ABC cân tại A, gọi O là trung điểm BC. Vẽ đường tròn tâm O tiếp xúc với
AB, AC tại H, K. Một tiếp tuyến đường tròn (O) cắt các cạnh AB, AC ở M và N.
a) Cho B C  . Tính MON ?
b) Chứng minh rằng OM, ON chia tứ giác BMNC thanh ba tam giác đồng dạng.
c) Cho BC = 2a, tính tích BM.CN
d) Tiếp tuyến MN ở vị trí nào thì tổng BM + CN nhỏ nhất?
HD tích không đổi nên tổng nhỏ nhất khi
Bài 65. Cho tam giác ABC nội tiếp (O), có góc C bằng 450
. Đường tròn đường kính AB cắt các
cạnh AC, BC lần lượt tại M và N.
a) Chứng minh rằng MN OC
b) Tính
AB
MN
HD tam giác MBC cân tại M nên MB = MC, mà OB = OC nên MO vuông góc với BC. Tương tự
suy ra O là trực tâm tam giác MBC. Đpcm
b) tứ giác MONB là hình thang cân nên OB = MN. Góc AOB = 2 góc C = 900
. 2
AB
MN

Bài 66. *Cho đoạn thẳng AB. Trên cùng một nửa mặt phẳng bờ AB vẽ nửa đường tròn (O)
đường kính AB và các tiếp tuyến Ax, By. Qua điểm M thuộc nửa đường tròn này, kẻ tiếp tuyến
cắt Ax, By thứ tự tại C và D. Gọi N là giao điểm của AD và BC. Chứng minh rằng MN vuông
góc với AB
Hd: Cm MN song song AC theo đảo định lí Talet
Bài 67. *Cho đường tròn tâm O, điểm K nằm bên ngoài đường tròn. Kẻ các tiếp tuyến KA, KB
với đường tròn (A, B là các tiếp điểm). Kẻ đường kính AOC. Tiếp tuyến đường tròn (O) tại C
cắt AB tại E. Chứng minh rằng:
a) Các tam giác KBC và OBE đồng dạng
b) CK vuông góc với OE
Hình học kì 1 nâng cao Thầy Hồng Trí Quang
25
Hd a)
BE OB
tan BEC tanOKB
BC KB
   ; b) Gọi I là giao điểm BC và OE, H là giao CK và OE.
Xét hai tam giác IBE và IHC ta có góc H vuông, đpcm
ĐƯỜNG TRÒN NỘI TIẾP TAM GIÁC
Đường tròn tiếp xúc với ba cạnh của tam giác gọi là đường tròn nội tiếp tam giác, còn tam giác
gọi là ngoại tiếp đường tròn. Tâm của đường tròn nội tiếp tam giâc là giao điểm ba đường phân
giác các góc trong của tam giác.
Tính chất 1: Cho tam giác ABC. Gọi D, E, F thưo thứ tự là các tiếp điểm của đường tròn nội tiếp
(O) với AB, AC, BC. Chứng minh rằng
AB AC BC
AD AE
2
 
 
Bài 68. Cho tam giác ABC (AC > AB) và trung tuyến AD. Các đường tròn nội tiếp tam giác
ABD và tam giác ADC tiếp xúc với AD tại E và K tương ứng. Chứng minh rằng: 𝐴𝐶 − 𝐴𝐵 =
2𝐸𝐾.
Bài 69. Cho tam giác ABC có ba cạnh tương ứng là a, b, c. Đường tròn (O) nội tiếp tam giác
và tiếp xúc cạnh AB tại D. Tính số đo góc C biết: AC.BC = 2AD.DB Đs vuông tại C
Trường hợp đặc biệt, bán kính trong tam giác vuông
Bài 70. Cho tam giác ABC vuông tại A Gọi S là diện tích tam giác, R là bán kính đường tròn
ngoại tiếp. Chứng minh R r 2S 
Từ đẳng thức 2r = b + c – 2R suy Đpcm
Tính chất 2: Ứng dụng diện tích
Bài 71. Cho tam giác ABC vuông tại A, AB = 5. Tính độ dài AC, BC biết rằng số đo chu vi
bằng số đo diện tích.
Từ gt
r
(a b c) a b c
2
     nên r = 2. Từ đó b + c – a = 2.r nên a = 13, b = 12.
Bài 72. Gọi a b ch ;h ;h là các đường cao tương ứng của một tam giác; r là bán kính đường tròn
nội tiếp. Chứng minh các bất đẳng thức sau
a) a b ch h h 9r  
b) 2 2 2
a b ch h h 27r  
Ta có a b c
1 1 1 1 1 1
h h h 2S r(a b c) 9r
a b c a b c
   
             
   
Hình học kì 1 nâng cao Thầy Hồng Trí Quang
26
2 2 2
a b c a b ch h h 3(h h h ) 27r     
Tính chất 3: Tỉ số phân giác và góc ở tâm 0A
BIC 90
2
 
Bài 73. Cho tam giác ABC vuông tại A Nếu AB = 9cm, AC = 12cm. Gọi I là tâm của đường
tròn nội tiếp, G là trọng tâm tam giác. Tính IG
HD Sử dụng định lí đảo talet, gọi BI cắt AC tại N thì
BI 2 BG
BN 3 BM
  nên IG//AC. Vậy IG = 1cm
Bài 74. Cho tam giác ABC vuông tại A (không cân). Gọi E là trung điểm của BC, I là tâm
đường tròn nội tiếp tam giác ABC, tam giác IEC vuông. Tính tỉ số giữa các cạnh của tam giác
ABC.
Hd: Chứng minh tam giác IEC vuông tại I; Gọi D là giao CI và AB; có góc BID bằng 45 suy ra
BIE = 45.
Suy ra tam giác BDI = BEI suy ra BC = 2 BE = 2BD
Áp dụng tính chất phân giác, ta có AC = 2AD; Từ đó BC + AC = 2AB.
Kết hợp với Pitago ta có
3 4 5
AC AB BC
 
Tự luyện
Bài 75. Cơ bản Cho tam giác ABC cân tại C, nội tiếp đường tròn tâm (O) bán kính R = 1. Cạnh
bên AC gấp hai lần cạnh đáy BC. Đường tròn tâm I nội tiếp trong tam giác. Tính bán kính của
đường tròn (I).
HD Sử dụng Pitago tính
15
4
CM  ;
Ta có 5CD CI ID r   ;
15
4 1;
8
OD r AD   . Vậy
3
8
r 
Hình học kì 1 nâng cao Thầy Hồng Trí Quang
27
Bài 76. Từ một điểm A ở bên ngoài đường tròn (O) ta vẽ hai tiếp tuyến AB, AC với đường tròn
(B, C là các tiếp điểm). Trên AO lấy điểm M sao cho AM = AB. Các tia BM và CM lần lượt cắt
đường tròn tại một điểm thứ hai là D và E. Chứng minh rằng:
1/ M là tâm đường tròn nội tiếp tam giác OBC
2/ DE là đường kính của đường tròn (O).
HD 1/ Chứng minh 𝐶𝐵𝑀̂ = 𝑂𝐵𝑀̂ cùng phụ với hai góc bằng nhau
2/ OE // BC (2 góc sole trong bằng nhau); tương tự OD // BC. Vậy E, O, D thẳng hàng.
Bài 77. Tam giác ABC vuông tại A ngoại tiếp đường tròn (I; r). Gọi G là trọng tâm tam giác.
Tính các cạnh của tam giác ABC theo r biết IG song song với AC
Bài 78. Cho tam giác ABC vuông tại A có đường cao AH. Gọi (O; r), (𝑂1; 𝑟1), (𝑂2; 𝑟2) theo
thứ tự là các đường tròn nội tiếp các tam giác ABC, ABH, ACH.
a) Chứng minh rằng: 𝑟 + 𝑟1 + 𝑟2 = 𝐴𝐻
b) Chứng minh rằng 𝑟2
= 𝑟1
2
+ 𝑟2
2
c) Tính 𝑂1 𝑂2 nếu biết AB = 3, AC = 4
HD a) Tính 𝑟 + 𝑟1 + 𝑟2 theo ba cạnh, b) Sử dụng tỉ số đồng dạng và tỉ lệ thức, c) Áp dụng pitago
có    1 2 1 2
2 22
1 2 r rOO r r  1 2 2OO 
Bài 79. Cho tam giác ABC có chu vi 80cm ngoại tiếp đường tròn (O). Tiếp tuyến của đường tròn
(O) song song với BC cắt AB, AC theo thứ tự tại M và N.
a) Cho biết MN = 9,6 cm. Tính độ dài BC
b) Cho biết AC – AB = 6cm, tính độ dài các đoạn AB, AC, BC để MN đạt giá trị lớn nhất.
HD AMN
ABC
PMN
BC P
 Đặt BC = x, suy ra BC = 24 hoặc BC = 16cm
b) 40.MN = x (40 – x) 2
(x 20) 400 400     nên max MN = 10 ; x = 20
Từ đó BC = 20, AB = 27, AC = 33.
Công thức: 𝑆 = 𝑃𝑟
Bài 80. *Cho tam giác ABC ngoại tiếp đường tròn (O). Gọi D, E, F theo thứ tự là tiếp điểm trên
cách cạnh BC, AB, AC. Gọi H là chân đường vuông góc kẻ từ D đến EF. Chứng minh rằng:
BHE CHF
Hd Kẻ BI, CK vuông với EF, chứng minh BEI đồng dạng CFK, tiếp đó BHI đồng dạng CHK
Hình học kì 1 nâng cao Thầy Hồng Trí Quang
28
Bài 81. *Cho tam giác ABC. Đường tròn (O) nội tiếp tam giác tiếp xúc với BC tại D. Vẽ đường
kính DN của đường tròn (O). Tiếp tuyến của đường tròn (O) tại N cắt AB, AC theo thứ tự tại I,
K.
a) Chứng minh rằng:
NI DC
NK DB

b) Gọi F là giao điểm của AN và BC. Chứng minh rằng BD = CF.
Hd a) tam giác NOI đồng dạng DBO nên 2
NI.DB r NK.DC 
b) Ta có
NK NI NK NI IK AK NF
DB DC DB DC BC AC FC

    

từ đó FC = DB
Bài 82. *Cho đường tròn (O) nội tiếp tam giác đều ABC. Một tiếp tuyến của đường tròn cắt
cạnh AB và AC theo thứ tự tại M và N.
a) Tính diện tích tam giác AMN biết BC = 8, MN = 3
b) Chứng minh rằng: 𝑀𝑁2
= 𝐴𝑀2
+ 𝐴𝑁2
− 𝐴𝑀. 𝐴𝑁
c) *Chứng minh rằng: 1
AM AN
MB NC
 
Bài 83. *Cho tam giác ABC có BC = a, AC = b, AB = c. Gọi (I) là đường tròn nội tiếp tam giác.
Đường vuông góc với CI tại I cắt AC, AB theo thứ tự tại M và N. Chứng minh rằng:
a) 2 2
AM.BN IM IN 
b)
2 2 2
IA IB IC
1
bc ca ab
  
Hd a) Tam giác AMI đồng dạng AIB, AIB đồng dạng INB nên AMI đồng dạng INB nên AM.BN
= IM . IN
Hình học kì 1 nâng cao Thầy Hồng Trí Quang
29
b) Đặt AM = m, BN = n, IM = IN = x. Do tam giác AMI và AIB đồng dạng nên AM.AB =
IA2
=mc
2
IA m
bc b
 . Tương tự
2
IB n
ca a

Theo Pitago 2 2 2
IC CM IM  (b m)(a n) mn    ab bn am  
Do đó
2 2 2
IC n m IA IB
1 1
ab a b bc ca
     
ĐƯỜNG TRÒN BÀNG TIẾP
Bài 84. Đtròn bàng tiếp *Gọi D, E, F là tâm các đường tròn bàng tiếp của tam giác ABC. Chứng
minh tam giác DEF là tam giác nhọn.
HD Gọi I là tâm nội tiếp, suy ra CE vuông CD; IA vuông IE nên AEC bằng AID mà AID bằng
nửa tổng A + C nhọn nên DEF nhọn
VỊ TRÍ TƯƠNG ĐỐI CỦA HAI ĐƯỜNG TRÒN
Hai đường tròn cắt nhau
Tính chất: Trung trực
Bài 85. Cho ba đường tròn 1 2 3; ;O O O có cùng bán kính R và cùng đi qua một điểm I. Gọi các
giao điểm khác I của ba đường tròn      1 2 2 3 3 11
; ; ; ; ;O O O O O O tương ứng là A, B, C. Chứng
minh rằng:
a) 1 2 3ABC OO O  
b) Đường tròn ngoại tiếp tam giác ABC cũng có bán kính R.
c) I là trực tâm tam giác ABC.
d) Các đường 3 2 1; ;AO BO CO đồng quy.
Bài 86. Cho hai đường tròn (O) và (O’) có cùng bán kính, cắt nhau tại A và B. Kẻ cát tuyến
chung DAE của hai đường tròn (D thuộc (O), E thuộc (O’)). Chứng minh rằng BD = BE.
Nxet: Bài này hơi khó, nếu học kì 2 sử dụng tứ giác nội tiếp sẽ đơn giản hơn.
Hình học kì 1 nâng cao Thầy Hồng Trí Quang
30
Gọi I là giao điểm AB và OO’, hạ OH, O’K, IM vuông góc với DE. Chứng minh tam giác HIK
cân do IM là trung tuyến đồng thời là đường cao. Do đó BDE cân.
Hai đường tròn tiếp xúc
Tính chất OO’ = R + R’
Bài 87. Cho hai đường tròn (O; R) và (O’; R’) tiếp xúc ngoài nhau tại M (R > R’). AB là tiếp
tuyến chung ngoài (A thuộc (O), B thuộc (O’)).
a) Tính độ dài AB theo R, R’ Đs 2 'AB RR
b) Gọi C là giao điểm của MB với (O). Chứng minh ba điểm A, O, C thẳng hàng
c) Tính độ dài MA, MB.
d) Nếu điểm M cố định, bán kính R, R’ không đổi. Hai điểm O, O’ thay đổi sao cho hai đường
tròn (O; R) và (O’; R’) tiếp xúc nhau tại M. Tìm quỹ tích trung điểm I của BC.
HD cách 1. Gọi M là điểm tiếp xúc, tiếp tuyến tại M cắt AB tại I thì AB = 2IM và
2
. 'IM OM O M
Cách 2. Từ O’ hạ O’H vuông góc với OA, sử dụng định lí Pitago.
Bài 88. Áp dụng bài trên
Cho hai đường tròn (O;R) và (O’;R’) tiếp xúc ngoài tại A. Gọi MN là tiếp tuyến chung ngoài.
Đường tròn (I;r) tiếp xúc với MN và tiếp xúc ngoài với hai đường tròn (O) và (O’). Chứng minh
rằng
1 1 1
'r R R
 
𝐻𝐷 𝑎) 𝑂′
𝑂2
= 𝑂′
𝐾2
+ 𝑂𝐾2
; 𝑏) 𝑀𝑁 = 𝑀𝐸 + 𝐸𝑁
Hai đường tròn không cắt nhau
Hình học kì 1 nâng cao Thầy Hồng Trí Quang
31
Bài 89. Cho hai đường tròn ngoài nhau (O) và (O’). Kẻ tiếp tuyến chung ngoài MM’và tiếp
tuyến chung trong NN’ của hai đường tròn (M, N là tiếp điểm của đường tròn (O); M’, N’ là tiếp
điểm của đường tròn (O’)). Gọi giao điểm của MM’ và NN’ là P, giao điểm của MN và M’N’ là
Q.
a) Tính góc OPO’
b) Gọi AB là tiếp tuyến chung ngoài thứ hai, NN’ cắt AB tại K. Chứng minh KN = PN’.
c) Chứng minh ba điểm O, Q, O’ thẳng hàng.
HD a) Phân giác của hai góc kề bù
b) Góc MQM’ vuông, chứng minh tổng hai góc còn lại bằng 900
. Sai lầm khi sử dụng đồng vị, cho
góc Q bằng góc O’.
'
OQ
OO
   
Tự luyện
Hai đường tròn cắt nhau
Bài 90. Cho điểm A nằm ngoài đường tròn (O). Vẽ đường tròn tâm A bán kính AO. Gọi CD là
tiếp tuyến chung của hai đường tròn, C thuộc (O), D thuộc (A). Đường nối tâm OA cắt đường
tròn (O) ở H. Chứng minh rằng DH là tiếp tuyến của (O).
Tiếp xúc
Bài 91. Cho hai đường tròn (O) và (O’) tiếp xúc ngoài tại A. Gọi AB là đường kính của đường
tròn (O); AC là đường kính của đường tròn (O’); DE là tiếp tuyến chung của hai đường tròn, D
thuộc (O), E thuộc (O’); K là giao điểm của BD và CD.
a) Tứ giác ADKE là hình gì?
b) Chứng minh AK là tiếp tuyến chung của hai đường tròn (O) và (O’).
Hình học kì 1 nâng cao Thầy Hồng Trí Quang
32
c) Gọi M là trung điểm BC. Chứng minh rằng MK vuông góc với DE.
Bài 92. Hai đường tròn (O;R) và (O’;R’) tiếp xúc ngoài tại A. Gọi BC, DE là các tiếp tuyến
chung của hai đường tròn (B, D thuộc đường tròn tâm (O)).
a) Chứng minh BDEC là hình thang cân
b) Tính diện tích hình thang cân đó.
Bài 93. Cho nửa đường tròn tâm O đường kính AB. Gọi OC là bán kính vuông góc với AB, d là
tiếp tuyến với nửa đường tròn tại C. Gọi (I) là đường tròn tiếp xúc trong với nửa đường tròn tâm
O và tiếp xúc với đường kính AB. Chứng minh rằng điểm I cách đều đường thẳng d và điểm O.
Hai đường tròn không giao nhau
Bài 94. Cho hai đường tròn (O) và (O’) ở ngoài nhau. Qua (O) kẻ các tia tiếp tuyến với đường
tròn (O’), chúng cắt (O) tại A, B. Qua (O’) kẻ các tiếp tuyến với (O) chung cắt (O’) tại C và D.
Chứng minh rằng A, B, C, D là bốn đỉnh của hình chữ nhật.
MỘT SỐ BÀI TOÁN VẼ THÊM ĐƯỜNG PHỤ
Vẽ đường kính vuông góc với một dây cung
Bài 95. Cho góc xOy có số đo bằng 600
. Lấy điểm E trên tia phân giác của góc đó. Vẽ đường
tròn (E) cắt tia Ox tại A và B, cắt tia Oy tại C và D sao cho OA < OB, OC < OD. Vẽ đường tròn
(F) đi qua ba điểm E, A, D. Chứng minh rằng:
1/ AB = CD
2/ 𝐴𝐸𝐷̂ = 1200
3/ F thuộc đường tròn (E)
HD 1/ Từ E hạ EH, EK vuông góc AB, CD, ta có EH = EK; 3/ Chứng minh tam giác AEF cân
tại F có góc ở đáy 600
nên đều. từ đó EA = EF.
Vẽ tiếp tuyến chung tại điểm tiếp xúc của hai đường tròn
Bài 96. Cho nửa đường tròn (O) đường kính AB. Vẽ bán kính OC vuông góc với AB (C thuộc
đường tròn). Từ C vẽ tiếp tuyến xy với nửa đường tròn. Vẽ đường tròn (K) tiếp xúc với AB và
tiếp xúc với (O). Chứng minh rằng điểm K luôn cách đều điểm O và đường thẳng xy.
HD Gọi M, N lần lượt là tiếp điểm của (K) với AB và (O). Từ N vẽ tiếp tuyến chung của hai
đường tròn cắt AB tại D và cắt MK tại E. Ta chứng minh E thuộc xy
DEM DON   nên EM = ON = OC suy ra tứ giác ECOM là h.b.h tức E thuộc xy
Từ EM = ON nên KE = KO, đpcm
Hình học kì 1 nâng cao Thầy Hồng Trí Quang
33
Bài 97. **Cho hai đường tròn (O) và (O’) cắt nhau tại A, B. Qua A kẻ hai cát tuyến cắt (O) tại C
và P, cắt (O’) tại D và Q tương ứng. Chứng minh rằng CD = PQ khi và chỉ khi AB là phân giác
của góc 𝑃𝐴𝐷̂
Luyện tập
Bài 98. Cho đường tròn (O; R) dây AB bất kì và tiếp tuyến Ax. Vẽ BH vuông góc với Ax.
Chứng minh rằng tỉ số
2
AB
BH
không đổi
HD Kẻ OK vuông AB thì 2
AB 2OA.BH
Bài 99. Cho tam giác ABC vuông cân tại A, BC 2 2cm . Vẽ các đường tròn (B; BA) và (C;
CA). Một đường thẳng bất kì đi qua A cắt đường tròn (B) và (C) lần lượt tại D và E. Chứng
minh rằng: 2 2
AD AE không đổi
HD Kẻ BH, CK vuông DE thì BH = AK nên 2 2
AD AE
2 2
2AD AE
4 4 4.AB 16
2 2
   
      
   
Bài 100. Cho hai đường tròn đồng tâm (O) có bán kính R và r (R > r). Gọi A là một điểm
cố định trên đường tròn nhỏ. Qua A vẽ đường thẳng xy cắt đường tròn lớn tại B và C, cắt đường
tròn nhỏ tại một điểm thứ hai là D
1/ Chứng minh rằng AB = CD
2/ Vẽ dây AM của đường tròn nhỏ sao cho AM vuông góc xy. Chứng minh rằng trọng tâm G
của tam giác MBC là một điểm cố định
3/ Chứng minh rằng khi xy xoay quanh A thì tổng: 2 2 2
AM AB AC  không đổi.
HD 1/ Kẻ OH vuông BC; 2/ G chia OA theo tỉ lệ 1:2 nên cố định
3/ 2 2 2
AM AB AC  2 2 2
(2OH) (HB AH) (HC HA)    
2 2 2 2
2(OH HB ) 2(OH HA )    2 2
2(R r ) 
Bài 101. *Cho đường tròn (O; 1). Lấy một điểm A cố định trên đường tròn. Vẽ tam giác
MAB vuông tại M, AB là một dây cung của đường tròn (O). Tìm giá trị lớn nhất của độ dài OM
HD Từ O hạ OH vuông AB thì HA = HB = HM = x. Xét ba điểm H, O, M ta có:
OM OH HM  2
1 x x      
2
2 2 2 2
1 1 1 x x 2
 
     
 
Dấu bằng xảy ra khi AB 2 và H, M, O thẳng hàng (tức tam giác AOB vuông tại O)
Hình học kì 1 nâng cao Thầy Hồng Trí Quang
34
Bài 102. Cho tam giác ABC vuông tại A, đường tròn (O) nội tiếp có bán kính r. Vẽ đường
thẳng d đi qua O cắt hai cạnh AB, AC lần lượt tại M và N. Xác định vị trí của d để tam giác
AMN có diện tích nhỏ nhất?
HD gọi S là diện tích tam giác AMN. Ta có:
2S = AM.AN
AOM AON
1
S S S r(AM AN)
2
   
1
.r.2. AM.AN
2
 r 2S
S r 2S 2
S 2r 
Dấu bằng xảy ra khi và chỉ khi AM = AN tức d vuông góc OA
Bài 103. Cho đường tròn (O; R). Từ một điểm A cách O là 2R vẽ hai tiếp tuyến AD, AE
với đường tròn (D, E là tiếp điểm). Trên cung nhỏ DE lấy điểm F. Qua F vẽ tiếp tuyến thứ ba
với đường tròn cắt các tia AD, AE lần lượt tại M, N. Qua O vẽ một đường thẳng vuông góc với
OA cắt các tia AD, AE lần lượt tại B và C.
1/ Chứng minh rằng ABC ACB MON 
2/ Chứng minh rằng: 2
BM.CN OB
3/ Xác định vị trí của điểm F để tổng BM + CN có giá trị nhỏ nhất?
HD Gọi 2 tiếp điểm là D và E; ta có:
1 180 A
MON DOE B C
2 2

   
2/ Tam giác MBO MON MBO OCN     2
BM.CN OB 
3/
2R 3
OB
3
 ;
4R 3
BM CN 2 BM.CN
3
  
Dấu bằng xảy ra khi F là giao điểm của AO với đường tròn.
Bài 104. Cho các đường tròn (O; R) và (O; R’) có bán kính khác nhau, tiếp xúc ngoài nhau
tại M. Vẽ các tiếp tuyến chung ngoài AB, CD với hai đường tròn trong đó A, D thuộc đường
tròn (O); B, C thuộc đường tròn (O’). Chứng minh rằng:
1/ Ba đường AB, CD, OO’ đồng quy
2/ AB + CD = BC + AD.
HD 1/ Gọi giao AB và CD cắt nhau tại K, thì O, O’ đều nằm trên đường phân giác AKB
Hình học kì 1 nâng cao Thầy Hồng Trí Quang
35
2/ Từ M kẻ tiếp tuyến chung của 2 đường tròn cắt AB, CD tại E và T. Theo t/c tiếp tuyến ta có:
EA = EM = EF; FM = FD = FC.
Theo t/c đường trung bình hình thang: AD BC 2EF AB DC   
Bài 105. Cho đoạn thẳng AB = 2a. Đường tròn (C) tiếp xúc với AB tại A, đường tròn (D)
tiếp xúc với AB tại B, hai đường tròn này thay đổi nhưng luôn thuộc cùng một nửa mặt phẳng
bờ AB và luôn tiếp xúc ngoài với nhau tại M.
1/ Hỏi điểm M di động trên đường nào?
2/ Chứng minh rằng: 2
AC.BD a
HD M nằm trên đường tròn (O; a); 2/ Ta có: 2 2
AC.BD CM.DM OM a  
Bài 106. *Cho đường tròn (O) và một điểm P trong đường tròn. Qua P kẻ cát tuyến APB
bất kì. Đường tròn tâm I qua A, P tiếp xúc với (O) tại A, đường tròn tâm J qua B và P tiếp xúc
với (O) tại B. Giao điểm thứ hai của hai đường tròn (I) và (J) là M. Chứng minh rằng đường
thẳng IJ luôn đi qua một điểm cố định khi cát tuyến APB thay đổi. Từ đó suy ra tập hợp các
điểm M nằm trên một đường tròn cố định.
Hd góc API = ABJ suy ra PI song song BO
Tương tự suy ra PIOJ là h.b.h hay IJ đi qua K là trung điểm OP
Vì PM vuông góc với JI nên M thuộc đường tròn đường kính OP

Mais conteúdo relacionado

Mais procurados

Tuyển tập 500 bài Bất Đẳng Thức cổ điển
Tuyển tập 500 bài Bất Đẳng Thức cổ điểnTuyển tập 500 bài Bất Đẳng Thức cổ điển
Tuyển tập 500 bài Bất Đẳng Thức cổ điển
Nguyễn Việt Long
 
Bo de thi lop 10 mon toan co dap an
Bo de thi lop 10 mon toan co dap anBo de thi lop 10 mon toan co dap an
Bo de thi lop 10 mon toan co dap an
Tommy Bảo
 
Một số bài hình 9 chọn lọc
Một số bài hình 9 chọn lọcMột số bài hình 9 chọn lọc
Một số bài hình 9 chọn lọc
Ngo Quang Viet
 
Chuyen de toan logic roi rac li thuyet to hop
Chuyen de toan logic  roi rac li thuyet to hopChuyen de toan logic  roi rac li thuyet to hop
Chuyen de toan logic roi rac li thuyet to hop
lephucduc06011999
 
bộ đề+ đáp án đề thi học sinh giỏi hình học 8
bộ đề+ đáp án đề thi học sinh giỏi hình học 8bộ đề+ đáp án đề thi học sinh giỏi hình học 8
bộ đề+ đáp án đề thi học sinh giỏi hình học 8
Jackson Linh
 
Chuyen de mon toan nguyen tac dirichlet
Chuyen de mon toan nguyen tac dirichletChuyen de mon toan nguyen tac dirichlet
Chuyen de mon toan nguyen tac dirichlet
Cảnh
 
9 phương pháp giải phương trình nghiệm nguyên
9 phương pháp giải phương trình nghiệm nguyên9 phương pháp giải phương trình nghiệm nguyên
9 phương pháp giải phương trình nghiệm nguyên
Thấy Tên Tao Không
 
Một số chuyên đề nâng cao đại số lớp 7
Một số chuyên đề nâng cao đại số lớp 7Một số chuyên đề nâng cao đại số lớp 7
Một số chuyên đề nâng cao đại số lớp 7
vukimhoanc2vinhhoa
 
Phep nghich dao __
Phep nghich dao  __Phep nghich dao  __
Phep nghich dao __
Duc Tung
 

Mais procurados (20)

Tuyển tập 500 bài Bất Đẳng Thức cổ điển
Tuyển tập 500 bài Bất Đẳng Thức cổ điểnTuyển tập 500 bài Bất Đẳng Thức cổ điển
Tuyển tập 500 bài Bất Đẳng Thức cổ điển
 
Bo de thi lop 10 mon toan co dap an
Bo de thi lop 10 mon toan co dap anBo de thi lop 10 mon toan co dap an
Bo de thi lop 10 mon toan co dap an
 
Một số bài hình 9 chọn lọc
Một số bài hình 9 chọn lọcMột số bài hình 9 chọn lọc
Một số bài hình 9 chọn lọc
 
Chuyen de toan logic roi rac li thuyet to hop
Chuyen de toan logic  roi rac li thuyet to hopChuyen de toan logic  roi rac li thuyet to hop
Chuyen de toan logic roi rac li thuyet to hop
 
bộ đề+ đáp án đề thi học sinh giỏi hình học 8
bộ đề+ đáp án đề thi học sinh giỏi hình học 8bộ đề+ đáp án đề thi học sinh giỏi hình học 8
bộ đề+ đáp án đề thi học sinh giỏi hình học 8
 
CHUYÊN ĐỀ QUAN HỆ VUÔNG GÓC - CHƯƠNG 3 HÌNH HỌC 11 (THẦYHOÀNG THÁI VIỆT)
CHUYÊN ĐỀ QUAN HỆ VUÔNG GÓC - CHƯƠNG 3 HÌNH HỌC 11 (THẦYHOÀNG THÁI VIỆT)CHUYÊN ĐỀ QUAN HỆ VUÔNG GÓC - CHƯƠNG 3 HÌNH HỌC 11 (THẦYHOÀNG THÁI VIỆT)
CHUYÊN ĐỀ QUAN HỆ VUÔNG GÓC - CHƯƠNG 3 HÌNH HỌC 11 (THẦYHOÀNG THÁI VIỆT)
 
Phương Tích - Trục Đẳng Phương
Phương Tích - Trục Đẳng PhươngPhương Tích - Trục Đẳng Phương
Phương Tích - Trục Đẳng Phương
 
Bất đẳng thức hình học
Bất đẳng thức hình họcBất đẳng thức hình học
Bất đẳng thức hình học
 
Đặt ẩn phụ giải phương trình chứa căn
Đặt ẩn phụ giải phương trình chứa cănĐặt ẩn phụ giải phương trình chứa căn
Đặt ẩn phụ giải phương trình chứa căn
 
De cuong hinh hoc lop 10 nc
De cuong hinh hoc lop 10 ncDe cuong hinh hoc lop 10 nc
De cuong hinh hoc lop 10 nc
 
72 hệ phương trình
72 hệ phương trình72 hệ phương trình
72 hệ phương trình
 
Chứng minh đẳng thức vectơ và phân tích vectơ
Chứng minh đẳng thức vectơ và phân tích vectơ Chứng minh đẳng thức vectơ và phân tích vectơ
Chứng minh đẳng thức vectơ và phân tích vectơ
 
Bo de hinh hoc thcs
Bo de hinh hoc thcsBo de hinh hoc thcs
Bo de hinh hoc thcs
 
Tuyển tập các bài Toán Hình học lớp 9 ôn thi vào 10
Tuyển tập các bài Toán Hình học lớp 9 ôn thi vào 10Tuyển tập các bài Toán Hình học lớp 9 ôn thi vào 10
Tuyển tập các bài Toán Hình học lớp 9 ôn thi vào 10
 
Chuyen de mon toan nguyen tac dirichlet
Chuyen de mon toan nguyen tac dirichletChuyen de mon toan nguyen tac dirichlet
Chuyen de mon toan nguyen tac dirichlet
 
9 phương pháp giải phương trình nghiệm nguyên
9 phương pháp giải phương trình nghiệm nguyên9 phương pháp giải phương trình nghiệm nguyên
9 phương pháp giải phương trình nghiệm nguyên
 
Một số chuyên đề nâng cao đại số lớp 7
Một số chuyên đề nâng cao đại số lớp 7Một số chuyên đề nâng cao đại số lớp 7
Một số chuyên đề nâng cao đại số lớp 7
 
Phep nghich dao __
Phep nghich dao  __Phep nghich dao  __
Phep nghich dao __
 
TUYỂN TẬP 19 CHUYÊN ĐỀ BỒI DƯỠNG HSG TOÁN 9 VÀ ÔN THI VÀO LỚP 10 TRƯỜNG CHUYÊN
TUYỂN TẬP 19 CHUYÊN ĐỀ BỒI DƯỠNG HSG TOÁN 9 VÀ ÔN THI VÀO LỚP 10 TRƯỜNG CHUYÊNTUYỂN TẬP 19 CHUYÊN ĐỀ BỒI DƯỠNG HSG TOÁN 9 VÀ ÔN THI VÀO LỚP 10 TRƯỜNG CHUYÊN
TUYỂN TẬP 19 CHUYÊN ĐỀ BỒI DƯỠNG HSG TOÁN 9 VÀ ÔN THI VÀO LỚP 10 TRƯỜNG CHUYÊN
 
TỔNG HỢP KIẾN THỨC TOÁN LỚP 2
TỔNG HỢP KIẾN THỨC TOÁN LỚP 2TỔNG HỢP KIẾN THỨC TOÁN LỚP 2
TỔNG HỢP KIẾN THỨC TOÁN LỚP 2
 

Semelhante a 9 hinh nang cao hk 1 dap an

On tap toan 7 hoc ki ii va de thi hoc ki ii
On tap toan 7 hoc ki ii va de thi hoc ki iiOn tap toan 7 hoc ki ii va de thi hoc ki ii
On tap toan 7 hoc ki ii va de thi hoc ki ii
Tu Em
 
35 bai-tap-he-thuc-luong-trong-tam-giac-co-huong-dan
35 bai-tap-he-thuc-luong-trong-tam-giac-co-huong-dan35 bai-tap-he-thuc-luong-trong-tam-giac-co-huong-dan
35 bai-tap-he-thuc-luong-trong-tam-giac-co-huong-dan
nhan nguyen
 
Chuyên ð hình không gian c ði_n
Chuyên ð  hình không gian c  ði_nChuyên ð  hình không gian c  ði_n
Chuyên ð hình không gian c ði_n
chanpn
 
Chuyen de hinh_hoc_khong_gian
Chuyen de hinh_hoc_khong_gianChuyen de hinh_hoc_khong_gian
Chuyen de hinh_hoc_khong_gian
Vinh Lưu
 
Chuyên đề 2 tích vô hướng của hai vecto và ứng dụng
Chuyên đề 2 tích vô hướng của hai vecto và ứng dụngChuyên đề 2 tích vô hướng của hai vecto và ứng dụng
Chuyên đề 2 tích vô hướng của hai vecto và ứng dụng
phamchidac
 

Semelhante a 9 hinh nang cao hk 1 dap an (20)

9 hinh nang cao htq
9 hinh nang cao htq9 hinh nang cao htq
9 hinh nang cao htq
 
Hệ thức lượng và tỉ số lượng giác
Hệ thức lượng và tỉ số lượng giácHệ thức lượng và tỉ số lượng giác
Hệ thức lượng và tỉ số lượng giác
 
Hd on tap cac dang toan 9 - ch 1
Hd on tap cac dang toan 9 - ch 1Hd on tap cac dang toan 9 - ch 1
Hd on tap cac dang toan 9 - ch 1
 
CHUYÊN ĐỀ HÌNH HỌC 8 CHƯƠNG 3 & ĐỀ KIỂM TRA 2018
CHUYÊN ĐỀ HÌNH HỌC 8 CHƯƠNG 3 & ĐỀ KIỂM TRA 2018CHUYÊN ĐỀ HÌNH HỌC 8 CHƯƠNG 3 & ĐỀ KIỂM TRA 2018
CHUYÊN ĐỀ HÌNH HỌC 8 CHƯƠNG 3 & ĐỀ KIỂM TRA 2018
 
CẨM NANG CHINH PHỤC HÌNH HỌC VÀO LỚP 10 NĂM 2023 HỆ THỨC LƯỢNG TRONG TÁC VUÔN...
CẨM NANG CHINH PHỤC HÌNH HỌC VÀO LỚP 10 NĂM 2023 HỆ THỨC LƯỢNG TRONG TÁC VUÔN...CẨM NANG CHINH PHỤC HÌNH HỌC VÀO LỚP 10 NĂM 2023 HỆ THỨC LƯỢNG TRONG TÁC VUÔN...
CẨM NANG CHINH PHỤC HÌNH HỌC VÀO LỚP 10 NĂM 2023 HỆ THỨC LƯỢNG TRONG TÁC VUÔN...
 
GIẢI TAM GIÁC TRONG TỌA ĐỘ PHẲNG
GIẢI TAM GIÁC TRONG TỌA ĐỘ PHẲNGGIẢI TAM GIÁC TRONG TỌA ĐỘ PHẲNG
GIẢI TAM GIÁC TRONG TỌA ĐỘ PHẲNG
 
9 tu giac noi tiep
9 tu giac noi tiep9 tu giac noi tiep
9 tu giac noi tiep
 
đề Cương ôn tập toán học kỳ i1 toán 7
đề Cương ôn tập  toán học kỳ i1 toán 7đề Cương ôn tập  toán học kỳ i1 toán 7
đề Cương ôn tập toán học kỳ i1 toán 7
 
On tap toan 7 hoc ki ii va de thi hoc ki ii
On tap toan 7 hoc ki ii va de thi hoc ki iiOn tap toan 7 hoc ki ii va de thi hoc ki ii
On tap toan 7 hoc ki ii va de thi hoc ki ii
 
Duong tron bttl phan 6 ct1, ct2
Duong tron bttl phan 6 ct1, ct2Duong tron bttl phan 6 ct1, ct2
Duong tron bttl phan 6 ct1, ct2
 
35 bai-tap-he-thuc-luong-trong-tam-giac-co-huong-dan
35 bai-tap-he-thuc-luong-trong-tam-giac-co-huong-dan35 bai-tap-he-thuc-luong-trong-tam-giac-co-huong-dan
35 bai-tap-he-thuc-luong-trong-tam-giac-co-huong-dan
 
Chuyên ð hình không gian c ði_n
Chuyên ð  hình không gian c  ði_nChuyên ð  hình không gian c  ði_n
Chuyên ð hình không gian c ði_n
 
Chuyen de hinh_hoc_khong_gian
Chuyen de hinh_hoc_khong_gianChuyen de hinh_hoc_khong_gian
Chuyen de hinh_hoc_khong_gian
 
CHUYÊN ĐỀ :TỌA ĐỘ PHẲNG - PHƯƠNG PHÁP VECTƠ
CHUYÊN ĐỀ :TỌA ĐỘ PHẲNG - PHƯƠNG PHÁP VECTƠCHUYÊN ĐỀ :TỌA ĐỘ PHẲNG - PHƯƠNG PHÁP VECTƠ
CHUYÊN ĐỀ :TỌA ĐỘ PHẲNG - PHƯƠNG PHÁP VECTƠ
 
Chuyên đề 2 tích vô hướng của hai vecto và ứng dụng
Chuyên đề 2 tích vô hướng của hai vecto và ứng dụngChuyên đề 2 tích vô hướng của hai vecto và ứng dụng
Chuyên đề 2 tích vô hướng của hai vecto và ứng dụng
 
Hinh chuong2
Hinh chuong2Hinh chuong2
Hinh chuong2
 
THAM SỐ HÓA TRONG TỌA ĐỘ PHẲNG
THAM SỐ HÓA TRONG TỌA ĐỘ PHẲNGTHAM SỐ HÓA TRONG TỌA ĐỘ PHẲNG
THAM SỐ HÓA TRONG TỌA ĐỘ PHẲNG
 
Hh10 c1a
Hh10 c1aHh10 c1a
Hh10 c1a
 
Thay khanh iwaz
Thay khanh iwazThay khanh iwaz
Thay khanh iwaz
 
Bodeontap toan7
Bodeontap toan7Bodeontap toan7
Bodeontap toan7
 

Mais de Hồng Quang

Mais de Hồng Quang (20)

10 de tang hsg quan huyen thay hong tri quang
10 de tang hsg quan   huyen thay hong tri quang10 de tang hsg quan   huyen thay hong tri quang
10 de tang hsg quan huyen thay hong tri quang
 
Pt co ban bttl phan 6 ct0, ct1, ct2
Pt co ban bttl phan 6 ct0, ct1, ct2Pt co ban bttl phan 6 ct0, ct1, ct2
Pt co ban bttl phan 6 ct0, ct1, ct2
 
Duong tron bttl phan 5 ct1, ct2
Duong tron bttl phan 5 ct1, ct2Duong tron bttl phan 5 ct1, ct2
Duong tron bttl phan 5 ct1, ct2
 
9 pp danh gia
9 pp danh gia9 pp danh gia
9 pp danh gia
 
9 can thuc nc
9 can thuc nc9 can thuc nc
9 can thuc nc
 
20 cach cm bdt nesbit
20 cach cm bdt nesbit20 cach cm bdt nesbit
20 cach cm bdt nesbit
 
9 drichle
9 drichle9 drichle
9 drichle
 
[Htq] toan 9 hsg tp hn dap an
[Htq] toan 9 hsg tp hn   dap an[Htq] toan 9 hsg tp hn   dap an
[Htq] toan 9 hsg tp hn dap an
 
9 [htq] de thi hsg 3 2 lopluyenthi
9 [htq] de thi hsg 3 2 lopluyenthi9 [htq] de thi hsg 3 2 lopluyenthi
9 [htq] de thi hsg 3 2 lopluyenthi
 
9 tu giac noi tiep htq
9 tu giac noi tiep htq9 tu giac noi tiep htq
9 tu giac noi tiep htq
 
9 03 de thi tet 2016
9 03 de thi tet 20169 03 de thi tet 2016
9 03 de thi tet 2016
 
9 tu giac noi tiep htq
9 tu giac noi tiep htq9 tu giac noi tiep htq
9 tu giac noi tiep htq
 
Dau hieu tich chung minh tu giac noi tiep htq
Dau hieu tich chung minh tu giac noi tiep   htqDau hieu tich chung minh tu giac noi tiep   htq
Dau hieu tich chung minh tu giac noi tiep htq
 
9 pt vo ti (co ban) htq
9 pt vo ti (co ban) htq9 pt vo ti (co ban) htq
9 pt vo ti (co ban) htq
 
Đồ thị hàm số - toán lớp 9
Đồ thị hàm số - toán lớp 9Đồ thị hàm số - toán lớp 9
Đồ thị hàm số - toán lớp 9
 
Ba dạng hệ phương trình cơ bản
Ba dạng hệ phương trình cơ bảnBa dạng hệ phương trình cơ bản
Ba dạng hệ phương trình cơ bản
 
9 phuong trinh nghiem nguyen htq
9 phuong trinh nghiem nguyen htq9 phuong trinh nghiem nguyen htq
9 phuong trinh nghiem nguyen htq
 
9 pt nghiem nguyen phan 2
9 pt nghiem nguyen phan 29 pt nghiem nguyen phan 2
9 pt nghiem nguyen phan 2
 
11 hinh on tap htq
11 hinh on tap htq11 hinh on tap htq
11 hinh on tap htq
 
8 phuong trinh nghiem nguyen htq
8 phuong trinh nghiem nguyen htq8 phuong trinh nghiem nguyen htq
8 phuong trinh nghiem nguyen htq
 

Último

26 Truyện Ngắn Sơn Nam (Sơn Nam) thuviensach.vn.pdf
26 Truyện Ngắn Sơn Nam (Sơn Nam) thuviensach.vn.pdf26 Truyện Ngắn Sơn Nam (Sơn Nam) thuviensach.vn.pdf
26 Truyện Ngắn Sơn Nam (Sơn Nam) thuviensach.vn.pdf
ltbdieu
 
xemsomenh.com-Vòng Tràng Sinh - Cách An 12 Sao Và Ý Nghĩa Từng Sao.pdf
xemsomenh.com-Vòng Tràng Sinh - Cách An 12 Sao Và Ý Nghĩa Từng Sao.pdfxemsomenh.com-Vòng Tràng Sinh - Cách An 12 Sao Và Ý Nghĩa Từng Sao.pdf
xemsomenh.com-Vòng Tràng Sinh - Cách An 12 Sao Và Ý Nghĩa Từng Sao.pdf
Xem Số Mệnh
 
Tử Vi Là Gì Học Luận Giải Tử Vi Và Luận Đoán Vận Hạn
Tử Vi Là Gì Học Luận Giải Tử Vi Và Luận Đoán Vận HạnTử Vi Là Gì Học Luận Giải Tử Vi Và Luận Đoán Vận Hạn
Tử Vi Là Gì Học Luận Giải Tử Vi Và Luận Đoán Vận Hạn
Kabala
 
bài tập lớn môn kiến trúc máy tính và hệ điều hành
bài tập lớn môn kiến trúc máy tính và hệ điều hànhbài tập lớn môn kiến trúc máy tính và hệ điều hành
bài tập lớn môn kiến trúc máy tính và hệ điều hành
dangdinhkien2k4
 

Último (20)

bài thi bảo vệ nền tảng tư tưởng của Đảng.docx
bài thi bảo vệ nền tảng tư tưởng của Đảng.docxbài thi bảo vệ nền tảng tư tưởng của Đảng.docx
bài thi bảo vệ nền tảng tư tưởng của Đảng.docx
 
Giáo trình xây dựng thực đơn. Ths Hoang Ngoc Hien.pdf
Giáo trình xây dựng thực đơn. Ths Hoang Ngoc Hien.pdfGiáo trình xây dựng thực đơn. Ths Hoang Ngoc Hien.pdf
Giáo trình xây dựng thực đơn. Ths Hoang Ngoc Hien.pdf
 
26 Truyện Ngắn Sơn Nam (Sơn Nam) thuviensach.vn.pdf
26 Truyện Ngắn Sơn Nam (Sơn Nam) thuviensach.vn.pdf26 Truyện Ngắn Sơn Nam (Sơn Nam) thuviensach.vn.pdf
26 Truyện Ngắn Sơn Nam (Sơn Nam) thuviensach.vn.pdf
 
xemsomenh.com-Vòng Tràng Sinh - Cách An 12 Sao Và Ý Nghĩa Từng Sao.pdf
xemsomenh.com-Vòng Tràng Sinh - Cách An 12 Sao Và Ý Nghĩa Từng Sao.pdfxemsomenh.com-Vòng Tràng Sinh - Cách An 12 Sao Và Ý Nghĩa Từng Sao.pdf
xemsomenh.com-Vòng Tràng Sinh - Cách An 12 Sao Và Ý Nghĩa Từng Sao.pdf
 
TUYỂN TẬP ĐỀ THI GIỮA KÌ, CUỐI KÌ 2 MÔN VẬT LÍ LỚP 11 THEO HÌNH THỨC THI MỚI ...
TUYỂN TẬP ĐỀ THI GIỮA KÌ, CUỐI KÌ 2 MÔN VẬT LÍ LỚP 11 THEO HÌNH THỨC THI MỚI ...TUYỂN TẬP ĐỀ THI GIỮA KÌ, CUỐI KÌ 2 MÔN VẬT LÍ LỚP 11 THEO HÌNH THỨC THI MỚI ...
TUYỂN TẬP ĐỀ THI GIỮA KÌ, CUỐI KÌ 2 MÔN VẬT LÍ LỚP 11 THEO HÌNH THỨC THI MỚI ...
 
Bài giảng môn Truyền thông đa phương tiện
Bài giảng môn Truyền thông đa phương tiệnBài giảng môn Truyền thông đa phương tiện
Bài giảng môn Truyền thông đa phương tiện
 
Access: Chuong III Thiet ke truy van Query.ppt
Access: Chuong III Thiet ke truy van Query.pptAccess: Chuong III Thiet ke truy van Query.ppt
Access: Chuong III Thiet ke truy van Query.ppt
 
xemsomenh.com-Vòng Thái Tuế và Ý Nghĩa Các Sao Tại Cung Mệnh.pdf
xemsomenh.com-Vòng Thái Tuế và Ý Nghĩa Các Sao Tại Cung Mệnh.pdfxemsomenh.com-Vòng Thái Tuế và Ý Nghĩa Các Sao Tại Cung Mệnh.pdf
xemsomenh.com-Vòng Thái Tuế và Ý Nghĩa Các Sao Tại Cung Mệnh.pdf
 
Bài học phòng cháy chữa cháy - PCCC tại tòa nhà
Bài học phòng cháy chữa cháy - PCCC tại tòa nhàBài học phòng cháy chữa cháy - PCCC tại tòa nhà
Bài học phòng cháy chữa cháy - PCCC tại tòa nhà
 
60 CÂU HỎI ÔN TẬP LÝ LUẬN CHÍNH TRỊ NĂM 2024.docx
60 CÂU HỎI ÔN TẬP LÝ LUẬN CHÍNH TRỊ NĂM 2024.docx60 CÂU HỎI ÔN TẬP LÝ LUẬN CHÍNH TRỊ NĂM 2024.docx
60 CÂU HỎI ÔN TẬP LÝ LUẬN CHÍNH TRỊ NĂM 2024.docx
 
Tử Vi Là Gì Học Luận Giải Tử Vi Và Luận Đoán Vận Hạn
Tử Vi Là Gì Học Luận Giải Tử Vi Và Luận Đoán Vận HạnTử Vi Là Gì Học Luận Giải Tử Vi Và Luận Đoán Vận Hạn
Tử Vi Là Gì Học Luận Giải Tử Vi Và Luận Đoán Vận Hạn
 
30 ĐỀ PHÁT TRIỂN THEO CẤU TRÚC ĐỀ MINH HỌA BGD NGÀY 22-3-2024 KỲ THI TỐT NGHI...
30 ĐỀ PHÁT TRIỂN THEO CẤU TRÚC ĐỀ MINH HỌA BGD NGÀY 22-3-2024 KỲ THI TỐT NGHI...30 ĐỀ PHÁT TRIỂN THEO CẤU TRÚC ĐỀ MINH HỌA BGD NGÀY 22-3-2024 KỲ THI TỐT NGHI...
30 ĐỀ PHÁT TRIỂN THEO CẤU TRÚC ĐỀ MINH HỌA BGD NGÀY 22-3-2024 KỲ THI TỐT NGHI...
 
Trắc nghiệm CHƯƠNG 5 môn Chủ nghĩa xã hội
Trắc nghiệm CHƯƠNG 5 môn Chủ nghĩa xã hộiTrắc nghiệm CHƯƠNG 5 môn Chủ nghĩa xã hội
Trắc nghiệm CHƯƠNG 5 môn Chủ nghĩa xã hội
 
30 ĐỀ PHÁT TRIỂN THEO CẤU TRÚC ĐỀ MINH HỌA BGD NGÀY 22-3-2024 KỲ THI TỐT NGHI...
30 ĐỀ PHÁT TRIỂN THEO CẤU TRÚC ĐỀ MINH HỌA BGD NGÀY 22-3-2024 KỲ THI TỐT NGHI...30 ĐỀ PHÁT TRIỂN THEO CẤU TRÚC ĐỀ MINH HỌA BGD NGÀY 22-3-2024 KỲ THI TỐT NGHI...
30 ĐỀ PHÁT TRIỂN THEO CẤU TRÚC ĐỀ MINH HỌA BGD NGÀY 22-3-2024 KỲ THI TỐT NGHI...
 
bài tập lớn môn kiến trúc máy tính và hệ điều hành
bài tập lớn môn kiến trúc máy tính và hệ điều hànhbài tập lớn môn kiến trúc máy tính và hệ điều hành
bài tập lớn môn kiến trúc máy tính và hệ điều hành
 
xemsomenh.com-Vòng Lộc Tồn - Vòng Bác Sĩ và Cách An Trong Vòng Lộc Tồn.pdf
xemsomenh.com-Vòng Lộc Tồn - Vòng Bác Sĩ và Cách An Trong Vòng Lộc Tồn.pdfxemsomenh.com-Vòng Lộc Tồn - Vòng Bác Sĩ và Cách An Trong Vòng Lộc Tồn.pdf
xemsomenh.com-Vòng Lộc Tồn - Vòng Bác Sĩ và Cách An Trong Vòng Lộc Tồn.pdf
 
Giới Thiệu Về Kabala | Hành Trình Thấu Hiểu Bản Thân | Kabala.vn
Giới Thiệu Về Kabala | Hành Trình Thấu Hiểu Bản Thân | Kabala.vnGiới Thiệu Về Kabala | Hành Trình Thấu Hiểu Bản Thân | Kabala.vn
Giới Thiệu Về Kabala | Hành Trình Thấu Hiểu Bản Thân | Kabala.vn
 
các nội dung phòng chống xâm hại tình dục ở trẻ em
các nội dung phòng chống xâm hại tình dục ở trẻ emcác nội dung phòng chống xâm hại tình dục ở trẻ em
các nội dung phòng chống xâm hại tình dục ở trẻ em
 
ĐỀ KIỂM TRA CUỐI KÌ 2 BIÊN SOẠN THEO ĐỊNH HƯỚNG ĐỀ BGD 2025 MÔN TOÁN 10 - CÁN...
ĐỀ KIỂM TRA CUỐI KÌ 2 BIÊN SOẠN THEO ĐỊNH HƯỚNG ĐỀ BGD 2025 MÔN TOÁN 10 - CÁN...ĐỀ KIỂM TRA CUỐI KÌ 2 BIÊN SOẠN THEO ĐỊNH HƯỚNG ĐỀ BGD 2025 MÔN TOÁN 10 - CÁN...
ĐỀ KIỂM TRA CUỐI KÌ 2 BIÊN SOẠN THEO ĐỊNH HƯỚNG ĐỀ BGD 2025 MÔN TOÁN 10 - CÁN...
 
Kiến thức cơ bản về tư duy số - VTC Net Viet
Kiến thức cơ bản về tư duy số - VTC Net VietKiến thức cơ bản về tư duy số - VTC Net Viet
Kiến thức cơ bản về tư duy số - VTC Net Viet
 

9 hinh nang cao hk 1 dap an

  • 1. Hình học kì 1 nâng cao Thầy Hồng Trí Quang 1 Nội dung chuyên đề hình học bao gồm 1. Hệ thức lượng trong tam giác vuông 2. Tỉ số lượng giác 3. Đường tròn a. Định nghĩa và tính chất của đường tròn b. Tiếp tuyến và phương pháp chứng minh tiếp tuyến c. Tính chất hai tiếp tuyến cắt nhau d. Đường tròn nội tiếp tam giác e. Vị trí tương đối của hai đường tròn HỆ THỨC LƯỢNG TRONG TAM GIÁC VUÔNG Cho tam giác ABC vuông tại A, đường cao AH. Chứng minh: 1) AC2 = CH . CB AB2 = BH . BC 2) AH2 = HB . HC 3) AH.BC= AB.AC 4) 2 2 2 1 1 1 AH AB AC   5) BC2 = AC2 + AB2 (Định lý Pi-ta-go) Bài 1. Cho hình vuông ABCD. Gọi E là một điểm nằm giữa A, B. Tia DE và tia CB cắt nhau ở F. Kẻ đường thẳng qua D vuông góc với DE, đường thẳng này cắt đường thẳng BC tại G. Chứng minh rằng: a) Tam giác DEG cân b) Tổng 2 2 2 1 1 1 DE DF DC   c) Một đường thẳng Ax thay đổi đi qua A sao cho Ax cắt đoạn DC tại M và cắt đường thẳng BC tại N. Chứng minh rằng tổng 2 2 1 1 AM AN  là không đổi. yx 6 4 H CB A
  • 2. Hình học kì 1 nâng cao Thầy Hồng Trí Quang 2 a) Ta có: 1 3D D (cùng phụ với 2D ) xét àADE v CDG  ta có :    1 3 0 ( ) . . 90 AD DC gt D D cmt ADE CDG g c g A C                DE DG DEG    cân tại D b) vì DE = DG 2 2 1 1 DE DG   ta có : 2 2 2 2 1 1 1 1 DE DF DG DF    Xét tam giác DGF vuông tại D, ta có : 2 2 2 1 1 1 CD DG DF   (định lý 4) c) Từ A kẻ đường thẳng vuông góc với AM, cắt BC tại P, chứng minh tương tự ta có: 2 2 2 1 1 1 AM AN a   không đổi Bài 2. Cho hình vuông ABCD. Trên cạnh BC lấy điểm M, trên cạnh CD lấy điểm N. Tia AM cắt đường thẳng CD tại K. Kẻ AI vuông góc với AK cắt CD tại I. Biết 𝑀𝐴𝑁̂ = 450 1. Chứng minh: IN = MN và MN = ND + BM 2. MN = 5 cm, CM - CN = 1 cm. Tính diện tích tam giác AMN. 3. Từ điểm O trong tam giác AIK kẻ OP, OQ, OR lần lượt vuông góc với IK, AK, AI ( P IK, QAK, R AI). Xác định vị trí điểm O để 222 OROQOP  nhỏ nhất khi O, M di động. Tìm giá trị nhỏ nhất đó. 3 2 1 G F E D C BA
  • 3. Hình học kì 1 nâng cao Thầy Hồng Trí Quang 3 Ta có )1(AIAMADIABM  và ID = BM Ta có MN = IN = ID + DN = BM + DN b) Do 2 2 25CM CN  và CM - CN = 1 CM = 4; CN = 3 MN = 5 Từ đó tính diện tích AMN qua phần bù. )(155.6. 2 1 . 2 1 2 cmMNAHS AMN  3. Từ giả thiết ta có AQOR là hình chữ nhật 222 )( 222 22222 ADAPOPOA OPOAOROQOP    222 OROQOP  nhỏ nhất khi O là trung điểm của AD. Bài 3. Cho tam giác ABC vuông tại A trung tuyến AM. a) Chứng minh rằng 1 2 AM BC b) 0 15ABC  . Chứng minh rằng: 2 4 .BC AB AC HD Kẻ đường trung tuyến AM, đường cao AH. Tính 1 1 2 4 AH AM BC  . Sử dụng diện tích tam giác ABC. c) Nếu tam giác ABC vuông tại A có trung tuyến BM, 0 15ABM  và 16ABCS  . Tính độ dài BM d) Nếu 0 75BAC  , đường cao CH thỏa mãn: 1 . 2 CH AB Chứng minh tam giác ABC cân. A B CD KI M H N
  • 4. Hình học kì 1 nâng cao Thầy Hồng Trí Quang 4 b) 21 8 8 ABCS BM  d) Kẻ BD vuông AC, DP vuông AB thì DP là đường trung bình trong tam giác ABD. Tự luyện Bài 4. Cho tam giác ABC có AB = 6cm, AC = 8cm, các đường trung tuyến BD và CE vuông góc với nhau. a) Tính độ dài BC. Đs 𝐵𝐶 = 2√5 b) Tính độ dài AB, AC Bài 5. Tính cạnh đáy BC của tam giác cân ABC biết đường cao tương ứng với cạnh đáy bằng 15,6cm và đường cao tương ứng với cạnh bên bằng 12cm. Đs BC = 13cm Bài 6. Cho M thuộc miền trong của hình chữ nhật ABCD. Chứng minh rằng: 𝑀𝐴2 + 𝑀𝐶2 = 𝑀𝐵2 + 𝑀𝐷2 HD kẻ qua M đt song song với AB cắt AD, BC tại E và F. Sử dụng Pitago Bài 7. Cho hai điểm A, B cố định và điểm M di động sao cho tam giác MAB có ba góc nhọn. Gọi H là trực tâm của tam giác MAB và K là chân đường cao vẽ từ M của tam giác MAB. Tìm GTLN của tích KH.KM./. H K A B M
  • 5. Hình học kì 1 nâng cao Thầy Hồng Trí Quang 5 Ta có: BKM   HKA (g.g)  . . BK KM BK KA KM KH HK KA    Mặt khác: BK.KA 2 2 2 4 BK KA AB       . Dấu “=” xảy ra khi BK = KA 2 . 4 AB KM KH  . Vậy max (KM.KH) = 2 4 AB khi BK = KA, tức là K là trung điểm của AB. Bài 8. Cho tam giác ABC, gọi I là tâm đường tròn nội tiếp tam giác. Qua I dựng đường thẳng vuông góc với IA cắt AB, AC tại M và N. Chứng minh rằng : a) 2 2 BM BI CN CI  b) BM.AC + CN.AB + AI2 = AB.AC a) I là tâm đường tròn nội tiếp  ABC  AI là phân giác A ; MN AI 0 1 1 90 ( ) 2 2 AMI A B C     1 1 2 2 AMI MIB B MIB C     BMI BIC    2 . BM BI BI BM BC BI BC    (1) Tương tự 2 .CI CN CB (2) Từ (1) và (2) suy ra 2 2 BM BI CN CI  b) Từ chứng minh trên BMI INC   B C A IM N
  • 6. Hình học kì 1 nâng cao Thầy Hồng Trí Quang 6 . . BM MI BM CN MI NI IN NC    Mà AMN cân IM = IN BM.CN = IM2 Xột  AIM có: AI2 + IM2 = AM2 2 2 2 2 2 2 . . IM AM AI BM CN AM AI AM AN AI         =(AB-BM)(AC- CN) – AI2 = AB.AC-AB.CN- BM.AC+BM.CN-AI2 BM.AC +CN.AB +AI2 = AB.AC (đpcm) TỈ SỐ LƯỢNG GIÁC CỦA GÓC NHỌN Cho tam giác ABC vuông tại A. Ta có tỉ số lượng giác của góc nhọn: sin AC B BC  cos AB B BC  tan AC B AB  cot AB B AC  Công thức cơ bản: 2 2 sin cos 1B B  tan .cot 1B B  sin cosB C và sin cosC B Công thức nâng cao 2 2 1 1 tan cos x   2 2 1 1 cot sin x   Diện tích tam giác (trong đó r là bán kính đtròn nội tiếp) 1 1 . . .sin . 2 2 S a h b c A p r   Bán kính ngoại tiếp R 2 sin sin sin a b c R A B C    sin 2 a A R  Bài 9. Cho tam giác ABC có 3 góc nhọn, vẽ đường cao AD và BE. Gọi H là trực tâm của tam giác ABC. Gọi a, b, c lần lượt là độ dài các cạnh BC, CA, AB của tam giác ABC a) Chứng minh: tanB.tanC = AD HD b) Chứng minh: 2 . 4 BC DH DA 
  • 7. Hình học kì 1 nâng cao Thầy Hồng Trí Quang 7 Ta có tanB = AD BD ; tanC = AD DC  tanB.tanC = 2 . AD BD DC (1) Xét 2 tam giác vuông ADC và BDH có DAC DBH vì cùng phụ với góc C nên ta có : AD BD ADC BDH DC DH     . .AD DH DB DC   2 . AD AD BD DC HD  (2) Từ (1) và (2)  tanB.tanC = AD HD . Theo câu a. ta có: 2 2 ( ) . . 4 4 DB DC BC DH DA DB DC     Bài 10. Cho tam giác ABC có 3 góc nhọn. Chứng minh rằng: bc aA 22 sin  K G H E D A B C x F M N A B C
  • 8. Hình học kì 1 nâng cao Thầy Hồng Trí Quang 8 Gọi Ax là tia phân giác góc A, kẻ BM; CN lần lượt vuông góc với Ax Ta có sin sin 2 A BM MAB AB   suy ra .sin 2 A BM c Tương tự .sin 2 A CN b do đó ( ).sin 2 A BM CN b c   Mặt khác ta luôn có: BM CN BF FC BC a     Nên ( ).sin 2 A b c a  sin 2 2 . A a a b c b c     Bài 11. Tính: sin2 150 + sin2 250 + sin2 350 + sin2 450 + sin2 550 + sin2 650 + sin2 750 = sin2 150 + sin2 250 + sin2 350 + sin2 450 + sin2 550 + sin2 650 + sin2 750 = (cos2 750 + sin2 750 ) + (cos2 650 + sin2 650 ) +(cos2 550 + sin2 550 ) + sin2 450 = 1 + 1 + 1 + 1 2 = 1 3 2 Bài 12. Cho tam giác ABC a) Tính 0 sin15 b) Nếu tam giác ABC vuông tại A thỏa mãn 2 4 .BC AB AC thì tam giác ABC có một góc bằng 0 15 c) Nếu 0 45BAC  , 0 75ABC  . Trên cạnh AB lấy điểm M sao cho 1 . 3 AM AB Tính ACM a) Tính tỉ số lượng giác, c) Kẻ MN, BD vuông AC, đặt AN = x. Tính MC, MN, NC theo x có: 2 4 .MC MN NC Tự luyện Bài 13. Cho hình vuông ABCD, biết M, N theo thứ tự là trung điểm BC, CD. a) Tính độ dài AM, AN, MN. b) Gọi K là giao điểm của AC và MN, tính AK
  • 9. Hình học kì 1 nâng cao Thầy Hồng Trí Quang 9 c) Gọi H là hình chiếu của M trên AN, tính MH. d) Tính cos 𝑀𝐴𝑁̂ Bài 14. a) *Cho biết sinx = 0,6. Tính cosx, tanx và cotx b) Tính 2 0 2 0 2 0 2 0 2 0 2 0 2 0 15 25 35 45 55cos cos cos cos c 65 7s co 5o ssin      + ta có: 2 2 2 2 sin cos 1 cos 1 sin 1 0,6 0,8           + sin 0,6 3 cos 0,8 4 tan ; cos 0,8 4 sin 0,6 3 cot             Bài 15. Góc nhọn của một hình thang cân bằng 600 , đường phân giác của góc nhọn này chia đường chéo của hình thang cân theo tỉ số 4:11 và chia đáy thành hai đoạn mà hiệu độ dài hai đoạn này bằng 6cm. a) Chứng minh rằng DC = AB + AD; b) Tính các cạnh đáy của hình thang HD b) DC = 66cm; AB = 42cm. Bài 16. Cho tam giác ABC vuông tại A có trọng tâm G; BD là phân giác góc B, GD vuông góc với AC. a) Gọi E là trung điểm AG, chứng minh rằng ED song song BC b) Tính các góc của tam giác ABC. HD b) chứng minh góc ACB = 300 Bài 17. Cho tam giác ABC vuông tại A có đường cao AH. Gọi D là điểm đối xứng với A qua B. Gọi E là điểm thuộc tia đối của tia HA sao cho HE = 2HA và I là trung điểm HE. Chứng minh rằng a) tan 𝐼𝐸𝐷̂ = tan 𝐻𝐶𝐸̂; b) Tam giác DEC vuông. HD AH = a, HB = b; tan = tan = 2b/a. Bài 18. Góc nhọn của một hình thang cân bằng 600 , đường phân giác của góc nhọn này chia đường chéo của hình thang cân theo tỉ số 4:11 và chia đáy thành hai đoạn mà hiệu độ dài hai đoạn này bằng 6cm. a) Chứng minh rằng DC = AB + AD; b) Tính các cạnh đáy của hình thang HD b) DC = 66cm; AB = 42cm.
  • 10. Hình học kì 1 nâng cao Thầy Hồng Trí Quang 10 Bài 19. Cho tam giác ABC vuông tại A có trọng tâm G; BD là phân giác góc B, GD vuông góc với AC. c) Gọi E là trung điểm AG, chứng minh rằng ED song song BC d) Tính các góc của tam giác ABC. HD b) chứng minh góc ACB = 300 Bài 20. Cho tam giác ABC vuông tại A có đường cao AH. Gọi D là điểm đối xứng với A qua B. Gọi E là điểm thuộc tia đối của tia HA sao cho HE = 2HA và I là trung điểm HE. Chứng minh rằng a) tan 𝐼𝐸𝐷̂ = tan 𝐻𝐶𝐸̂; b) Tam giác DEC vuông. HD AH = a, HB = b; tan = tan = 2b/a. Bài 21. Tính 0 0 sin22 25';tan22 25' mà không dùng bảng số, không dùng máy tính? Bài 22. Cho tam giác ABC vuông tại A có đường cao AH và 2 4 .AH AM AN , trong đó M, N theo thứ tự là chân đường vuông góc hạ từ H đến AC, AB. Tính số đo các góc tam giác ABC. HD góc 150 ; 750 Bài 23. Cho tam giác ABC vuông tại A có BE là phân giác trong, I là tâm đường tròn nội tiếp. Biết 3 1 3 1 BI EI    . Tính số đo góc ACB? HD đặt tỉ số BI x EI  , tính được 2 4 .BE AB AE nên 0 60ACB  Luyện tập Bài 24. Ứng dụng hệ thức 0 tan15
  • 11. Hình học kì 1 nâng cao Thầy Hồng Trí Quang 11 B E H F C A a) Chứng minh rằng:  0 6 4 sin15 2 b) Tính 0 tan15 c) Cho tam giác ABC cân tại B,  0 30BAC . Trên cạnh BC lấy điểm D sao cho  2 2 AC BD . Tính số đo góc CAD? Hướng dẫn: a) Dựng tam giác ABC vuông tại A, có  0 30B . Dựng phân giác BD, khi đó:  0 sin15 sinABD AD BD b) Đs      0 6 4 tan15 2 3 6 4 c) Kẻ AH vuông BC, đặt AH = x. Tính được DH = x nên  0 60DAC Bài 25. Cho tam giác ABC vuông ở A, AH  BC, HE  AB, HF  AC ( H  BC, E  AB, F  AC). a. Chứng minh rằng: AE.AB = AF.AC; BH = BC.cos2 B. b. Chứng minh rằng: 3 3 AB BE CFAC  . c. Chứng minh rằng: 33 32 2 2 BC CF BE  . d. Cho BC = 2a. Tìm giá trị lớn nhất của diện tích tứ giác AEHF. a. AHB vuông tại H, có HE  AB nên AH2 = AB.AE. (1) Tương tự: AH2 = AC.AF (2). Từ (1) và (2) suy ra: AB.AE = AC.AF.
  • 12. Hình học kì 1 nâng cao Thầy Hồng Trí Quang 12 * BH = AB.cosB; AB = BC.cosB Suy ra BH = BC.cos2 B. b.(1,5đ). Áp dụng hệ thức lượng trong tam giác vuông: AB2 = BC.BH; AC2 = BC.CH; BH2 = AB.BE; CH2 = AC.CF nên 2 4 2 3 2 4 2 3 . . . AB BH AB BH AB BE AB BE CH AC CF CFAC AC CH AC       a. (1,5đ) Ta có BE = BH.cosB; BH = AB.cosB; AB = BC.cosB; Do đó: BE = AB.cos2 B = BC.cos3 B  BE2 = BC2 .cos6 B  3 32 2 2 .cos .BE BC B Tương tự ta có: 3 32 2 2 .sin .CF BC B  3 3 3 32 2 2 2 2 2 .(cos sin .)BE CF BC B B BC    b. (1,0đ) Ta có: SAEHF = AE.AF Lại có: 2 AH AE AB  Tương tự: 2 AH AF AC  Do đó: 4 4 3 3 3 2 . . . 2 2 AEHF AH AH AH AO a a S AB AC BC AH BC BC a       Max SAEHF = 2 2 a  ABC vuông cân tại A. Bài 26. *Cho tam giác ABC cân tại A có 0ˆA 20 ;AB AC b;BC a    . Chứng minh rằng: a3 + b3 = 3ab2
  • 13. Hình học kì 1 nâng cao Thầy Hồng Trí Quang 13  ABC cân tại A có góc BAC = 200 nên ABC = ACB = 800 Trên cạnh AC lấy D sao cho ABD = 600 , khi đó DBC = 200 nên BDC = 800   BDC cân tại B  BD = BC = a .  BDC  ABC ( g – g)  DC BC BC AC   DC = 2 a b  AD = b - 2 a b  BDE vuông có 0 60EBD  nên BE = 1 2 BD = 1 2 a và DE = BD 3 2 = a. 3 2 ; AE = b - 1 2 a. Áp dung định lý Pi-ta-go trong tg vuông ADE có : AD2 = AE2 + DE2  (b - 2 a b )2 = (b - 1 2 a)2 + (a. 3 2 )2  b2 - 2a2 + 4 2 a b = b2 - ab + 2 4 a + 2 3 4 a  4 2 a b = 3a2 –ab  a4 = 3a2 b2 - ab3  a4 + ab3 = 3a2 b2  a3 + b3 = 3ab2 Tự luyện Bài 27. Tính cos 36, cos 72 mà không dùng bảng số, không dùng máy tính? HD vẽ tam giác cân ABC tại A có 𝐴 = 360 , phân giác CD, đường cao DH, AD = DC = CB = 1; 0 os36 D AH c A  0 1 5 os36 4 c  0 5 1 os72 4 c Bài 28. Cho tam giác ABC có ba góc nhọn và có độ dài 3 cạnh BC, AC, AB lần lượt bằng a, b, c. Chứng minh rằng: ))((.. SinCSinBSinAcbaSinCcSinBbSinAa  Vẽ đường cao AH. Ta có SinC c SinB b HC AH SinC HB AH BSin  ...; Tương tự, suy ra: 0    k SinCSinBSinA cba SinC c SinB b SinA a Vậy kSinCSinBSinAcSinCbSinBaSinA ).(...  (1) Và (a + b + c) = (SinA + Sin B + SinC).k B C A D E
  • 14. Hình học kì 1 nâng cao Thầy Hồng Trí Quang 14 Suy ra: kSinCSinBSinASinCSinBSinAcba ).())((  (2) Từ (1) và (2) ta có đpcm Bài 29. Cho tam giác ABC nhọn, đường cao AH. Điểm M thuộc đoạn thẳng BC. Kẻ MK vuông góc AB, ML vuông góc AC ( K thuộc AB, L thuộc AC ). Đường thẳng qua A và vuông góc với AM cắt MK, ML thứ tự tại E, F. Từ B kẻ đường thẳng vuông góc với CE, cắt Ah tại I. Chứng minh rằng: a) Tam giác AIB đồng dạng với tam giác MCE b) EM ML BM AI và FM KM FM AC   c) Ba đường thẳng AH, BF, CE đồng quy. Bài 30. Cho tam giác ABC có đường phân giác AD, đường cao BH, đường trung tuyến CE đồng quy tại O. Vẽ EF vuông góc với BH (FBH). Chứng minh: a) CH.AE = EF.AC b) . os .cos . osAC c BAC BD ACB CDc ACB  Bài 31. Cho tam giác ABC có 0 15CAB  ; 0 30ABC  . Gọi M là trung điểm cạnh AB a) Tính số đo góc ACM b) Chứng minh rằng: . 2 AB BC CM AC  HD kẻ MN, BD vuông góc AC.Đặt CD = BD = x, đs 0 30MCA  Bài 32. Trên cạnh BC của tam giác ABC lấy điểm P sao cho PC = 2PB. Tính số đo góc ACB nếu 0 45ABC  , 0 60CPA  HD Kẻ PQ, CH vuông AB, đặt QB = QP = x. Đs 0 75BCA  ĐƯỜNG TRÒN Bài 1. Định nghĩa tính đối xứng của đường tròn Bài 33. Cho tam giác ABC nhọn, vẽ đường tròn tâm O đường kính BC cắt các cạnh AB, AC theo thứ tự tại D và E a) Chứng minh rằng CD vuông góc với AB; BE vuông góc với AC b) Gọi K là giao điểm của BE và CD. Chứng minh rằng: AK vuông góc với BC LG
  • 15. Hình học kì 1 nâng cao Thầy Hồng Trí Quang 15 a) Theotính chất tam giác BCD và tam giác BCE có cạnh BC là đường kính => tam giác BCD vuông tại D (=> CD vuông góc với AB) và tam giác BCE vuông tại E (=> BE vuông góc với AC) b) Xét tam giác ABC, ta có : à BE AC CD AB m BE CD K         K là trực tâm của tam giác ABC => AK vuông góc với BC Bài 34. *Cho tam giác cân ABC (AB = AC). Gọi H là trực tâm tam giác, gọi D, M, N lần lượt là trung điểm của BC, AB, CH. Chứng minh rằng trung điểm của MN là tâm đường tròn ngoại tiếp tam giác DMN. Bài 35. *Cho tam giác nhọn ABC nội tiếp đường tròn (O; R). Gọi M là trung điểm BC. Giả sử O nằm trong tam giác AMC hoặc O nằm giữa A và M. Gọi I là trung điểm AC. Chứng minh rằng: a) Chứng minh MA + MC > OA + OC b) Chu vi tam giác IMC lớn hơn 2R c) Chu vi tam giác ABC lớn hơn 4R HD IM + IC + MC = IM + IA + IC > MA + MC > OA + OC = 2R Bài 36. *Cho hai đường thẳng xy và x’y’ vuông góc nhau tại O. Một đoạn thẳng AB=6cm chuyển động sao cho A luôn nằm trên xy và B trên x’y’. Hỏi trung điểm M của AB chuyển động trên đường nào? Bài 37. *Cho ba điểm A, B, C bất kì và đường tròn (O) bán kính 1. Chứng minh rằng tồn tại điểm M nằm trên đường tròn (O) sao cho: MA + MB + MC ≥ 3. Hd. Sử dụng nguyên lí Drichle, gọi đường kính DE và chỉ ra (𝐷𝐴 + 𝐷𝐵 + 𝐷𝐶) + (𝐸𝐴 + 𝐸𝐵 + 𝐸𝐶) ≥ 3 Bài 2. Tính chât đối xứng của đường tròn K E D O CB A
  • 16. Hình học kì 1 nâng cao Thầy Hồng Trí Quang 16 1. Đường kính vuông góc dây cung thì đi qua trung điểm và ngược lại. 2. Hai dây cung bằng nhau khi và chỉ khi chúng cách đều tâm. 3. Dây cung nào gần tâm hơn thì dài hơn và ngược lại. 4. Số đo của cung bằng số đo góc ở tâm 5. Hai đường thẳng song song cắt đường tròn tạo thành hai cung có số đo bằng nhau Bài 38. Cho đường tròn tâm O đường kính AB = 13cm. Dây CD có độ dài 12cm vuông góc với AB tại H. a) Tính các độ dài đoạn HA, HB b) *Gọi M, N theo thứ tự là hình chiếu của H trên AC, BC. Tính diện tích tứ giác CMHN. HD b) ABCS 39 ; 2 2 CHN ABC S CH 6 S AH 13              CHN ABC 36 108 S .S 169 13    ; 2 CMHN 8 S 16 cm 13  Bài 39. Cho tam giác ABC nội tiếp đường tròn (O) có D, E, F theo thứ tự là trung điểm của BC, AC, AB. a) Gọi H là trực tâm của tam giác. Chứng minh hai tam giác HAB và ODE đồng dạng b) Chứng minh 1 OD / /AH,OD AH 2  c) Kẻ các đường thẳng DD’, EE’, FF’ sao cho DD’//OA, EE’//OB, FF’//OC. Chứng minh rằng các đường DD’, EE’, FF’ đồng quy. Bài 40. Cho nửa đường tròn tâm O đường kính AB, dây CD. Gọi H, K là chân các đường vuông góc kẻ từ A, B đến CD. Gọi I là trung điểm CD a) Chứng minh rằng IH = IK b) Chứng minh rằng CH = DK b) *Chứng minh rằng: AHKB ACB ADBS S S  c) Bết AB = 30cm, CD = 18cm, tính OI và diện tích lớn nhất của tứ giác AHKB. Hd a) gọi I là trung điểm CD b) Qua I kẻ các đường thẳng song song với AB, khi đó AHKB AEFBS S c) ta có IO = 12cm, AHKBS AB.II' AB.IO 30.12 360   
  • 17. Hình học kì 1 nâng cao Thầy Hồng Trí Quang 17 Bài 41. Cho tam giác cân ABC (AB = AC) nội tiếp đường tròn (O). Gọi M là trung điểm của cạnh AB, G là trọng tâm tam giác AMC. a) Chứng minh OA vuông góc với MG b) Chứng minh EG song song với AB c) Chứng minh 𝑂𝐺 ⊥ 𝑀𝐶. Bài 42. Cho tam giác ABC, các đường cao AD, BE, CF. Đường tròn tâm (O) đi qua D, E, F cắt BC, CA, AB theo thứ tự tại M, N, P. Kẻ MM’, NN’, PP’ lần lượt vuông góc với BC, CA, AB. a) Gọi I là giao điểm của MM’ và NN’, chứng minh O là trung điểm HI b) Chứng minh ba đường MM’, NN’, PP’ đồng quy Hd; Gọi O’ là trung điểm của HI suy ra O thuộc trung trực của dây DM, O cũng thuộc trung trực của dây EN. Do đó O’ cũng là tâm đường tròn đi qua sáu điểm D, M, N, E, F, P; và điểm I là điểm đối xứng với H qua O Chứng minh tương tự MM’ cắt PP’ tại I’ thì I’ cũng đxứng với H qua O. Do đó I trùng I’ Vậy 3 đt MM’, NN’, PP’ đồng quy tại I Vấn đề 3: vị trí tương đối giữa đường thẳng d và đường tròn. (O; R) 1. Cho đường tròn (O; R) và đường thẳng d khi đó có các trường hợp sau: 1.1. Nếu d(O;Δ) = OH > R thì đường thẳng và đường tròn không có điểm chung. Ta nói đường thẳng và đường tròn ngoài nhau hoặc không cắt nhau. 1.2. Nếu d(O; Δ) = OH = R khi đó đường thẳng và đường tròn có một điểm chung duy nhất chính là H. Khi đó ta nói đườngthẳng tiếp xúc đường tròn (đường thẳng này gọi là tiếp tuyến của (O)). 1.3.Nếu d(O; Δ) = OH < R thì đường thẳng d cắt đường tròn (O; R) tại hai điểm phân biệt A và B. Đường thẳng này gọi là cát tuyến với (O; R). 2. Vậy muốn xác định vị trí của đường thẳng d và đường tròn ta cần tìm bán kính R và khoảng cách d(O; d) rồi so sánh và kết luận. Bài 43. Cho Δcân OAB có OA=OB=5cm và AB=6cm. Hỏi bán kính R của đường tròn (O; R) có giá trị bao nhiêu để đường tròn tiếp xúc AB. Bài 44. Cho nửa đường tròn tâm O bán kính R, đường kính AB, M là một điểm thuộc nửa đường tròn. Qua M vẽ tiếp tuyến với nửa đường tròn. Gọi D và C theo thứ tự là các hình chiếu của A và B trên tiếp tuyến ấy. a) Chứng minh M là trung điểm CD
  • 18. Hình học kì 1 nâng cao Thầy Hồng Trí Quang 18 b) Chứng minh AB = BC + AD c) Giả sử AOM BOM , gọi E là giao điểm của AD với nửa đường tròn. Xác định dạng của tứ giác BCDE. d) Xác định vị trí điểm M trên nửa đường tròn ấy sao cho tứ giác ABCD có diện tích lớn nhất. Tính diện tích đó theo R Bài 45. Cho nửa đường tròn tâm O đường kính AB, Ax là tiếp tuyến của nửa đường tròn, C là một điểm thuộc nửa đường tròn, H là hình chiếu của C trên AB. Đường thẳng qua O vuông góc với AC cắt Ax tại M. Gọi I là giao điểm của MB và CH. a) Gọi N là giao điểm của BC và Ax, chứng minh MA = MN. b) Chứng minh I là trung điểm của CH. Tự luyện Bài 46. Cho đường tròn tâm O có đường kính AB = 2R. Gọi d là tiếp tuyến của đường tròn, A là tiếp điểm. Gọi M là điểm bất kì thuộc d. Qua O kẻ đường thẳng vuông góc với BC, cắt d tại N. a) Chứng minh tích AM.AN không đổi khi M chuyển động trên d b) Tìm giá trị nhỏ nhất của MN Bài 47. Cho nửa đường tròn tâm O đường kính AB. Vẽ các tiếp tuyến Ax, By với nửa đường tròn. Trên nửa đường tròn lấy điểm C. Các tia BC và AC lần lượt cắt Ax, By tại D và E. Gọi M, N theo thứ tự là trung điểm của AD và BE. a) Chứng minh rằng MN là tiếp tuyến của đường tròn (O) b) Vẽ CH vuông góc với AB, chứng minh ba đường CH, AN, BM đồng quy HD a) Chứng minh M, C, N thẳng hàng (tổng hai góc vuông); b) Chứng minh cùng đi qua trung điểm K của CH. Bài toán Chứng minh đường thẳng d là tiếp tuyến(O; R) Cách 1. Xác định H thuộc d sao cho OH d và chứng minh OH = R Cách 2. Xác định H thuộc d sao cho OH = R và chứng minh OH d Bài 48. Cho hình thang vuông ABCD tại A và D, có 0 90BMC với M là trung điểm AD. a) Cách 2. Chứng minh rằng AD là tiếp tuyến của đường tròn đường kính BC b) Cách 1. Chứng minh rằng BC là tiếp tuyến của đường tròn đường kính AD.
  • 19. Hình học kì 1 nâng cao Thầy Hồng Trí Quang 19 Bài 49. Cho nửa đường tròn tâm O đường kính AB, C là một điểm thuộc nửa đường tròn, H là hình chiếu của C trên AB. Qua trung điểm M của CH kẻ đường vuông góc với OC, cắt nửa đường tròn tại D và E. a) Chứng minh rằng CO.CK = CM.CH b) OC cắt DE ở K và cắt (O) tại I. Chứng minh rằng 2 2 .CE COCK c) Chứng minh AB là tiếp tuyến của đường tròn tâm C bán kính CD. Kí hiệu (C;CD) Bài 50. Cho tam giác ABC vuông tại A (AB < AC), đường cao AH. Gọi E là điểm đối xứng với B qua H. Đường tròn có đường kính EC cắt AC tại K. Chứng minh rằng HK là tiếp tuyến của đường tròn HD Gọi O là trung điểm EC, M là trung điểm AK. Chứng minh 0 90OKH Hai Tiếp tuyến cắt nhau Bài 51. Cho nửa đường tròn (O ; R) đường kính AB. Gọi Ax, By là các tia vuông góc với AB (Ax, By và nửa đường tròn cùng thuộc nửa mp có bờ là AB). Lấy M thuộc Ax, qua M kẻ tiếp tuyến với nửa đường tròn, cắt By tại N a) Tính góc MON b) CMR: MN = AM + BN c) CMR tích AM.BN không đổi d) Tứ giác AMNB có diện tích nhỏ nhất khi nào? Tính diện tích đó LG a) - theo tc của 2 tiếp tuyến cắt nhau, ta có: 1 2 3 4 1 ; 2 1 ; 2 O O AOH MA MH O O BOH NB NH       (1) - ta có:   0 0 2 3 1 1 .180 90 2 2 MON O O AOH BOH      b) do MN = MH + NH (2) => từ (1) và (2) : MN = MA + NB 4 32 1 y x H N M R BA O
  • 20. Hình học kì 1 nâng cao Thầy Hồng Trí Quang 20 c) Xét tam giác MON vuông tại O, theo hệ thức về cạnh và đường cao trong tam giác vuông, ta có : 2 2. . . à OH MH NH AM BN AM BN R m OH R       Bài 52. Cho hình vuông ABCD có cạnh bằng a. Với tâm B và bán kính a, vẽ cung AC nằm trong hình vuông. Qua điểm E thuộc cung đó, vẽ tiếp tuyến với cung AC, cắt DA và DC theo thứ tự tại M và N. a) Tính chu vi tam giác DMN b) Tính số đo góc MBN c) Chứng minh rằng: 2a MN a 3   Hd Chu vi = 2a, b) góc 450 , c) MN + MN < DM + DN + MN = 2a; MN + MN + MN > DM + DN + MN Bài tập nâng cao Bài 53. Cho đường tròn (O), điểm K bên ngoài đường tròn. Kẻ các tiếp tuyến KA, KB với đường tròn (A, B là các tiếp điểm). Kẻ đường kính AOC. Tiếp tuyến của đường tròn (O) tại C cắt AB ở E. Chứng minh rằng: a) Tam giác KBC đồng dạng với tam giác OBE b) CK vuông góc với OE vd 15 – 103. Từ Toán phát triển 73 – 79 (105) Bài 54. Cho đoạn thẳng AB. Vẽ về một phía của AB các tia Ax, By song song với nhau a) Dựng đường tròn (O) tiếp xúc với đoạn thẳng AB và tiếp xúc với các tia Ax, By b) Tính góc AOB c) Gọi các tiếp điểm của (O) với Ax, By, AB theo thứ tự là M, N, H. Chứng minh rằng MN là tiếp tuyến của đường tròn đường kính AB. d) Các tia Ax, By có vị trí như thế nào để HM = HN.
  • 21. Hình học kì 1 nâng cao Thầy Hồng Trí Quang 21 Bài 55. Cho hình thang vuông ABCD (vuông tại A, D), tia phân giác góc C đi qua trung điểm I của AD. a) Chứng minh rằng BC là tiếp tuyến của đường tròn (I; IA). b) Cho AD = 2a. Tính tích của AB.CD theo a c) Gọi H là tiếp điểm của BC với (I) nói trên. K là giao điểm của AC và BD. Chứng minh rằng HK song song với DC. Bài 56. Cho đường tròn tâm (O) đường kính AB, D là một điểm nằm trên đường tròn. Các tiếp tuyến của đường tròn tại A, D cắt nhau ở C. Gọi E là hình chiếu của D trên AB, gọi I là giao điểm của BC và DE. Chứng minh rằng DI = IE. Bài 57. Cho tam giác ABC cân tại A, O là trung điểm của BC. Vẽ đường tròn tâm O tiếp xúc với AB, AC tại H, K. Một tiếp tuyến với đường tròn (O) cắt các cạnh AB, AC ở M, N. a) Cho B C   . Tính MON b) Chứng minh rằng OM, ON chia tứ giác BMNC thành ba tam giác đồng dạng. c) Cho BC = 2a. Tính tích BM.CN d) Tiếp tuyến MN ở vị trí nào để tổng BM + CN nhỏ nhất Bài 58. Cho tam giác ABC vuông tại A, đường cao AH, HB = 20cm, HC = 45cm. Vẽ đường tròn tâm A bán kính AH. Kẻ các tiếp tuyến BM, Cn với đường tròn (M, N là tiếp điểm, khác H). a) Tính diện tích tứ giác BMNC b) Gọi K là giao điểm của CN và HA. Tính độ dài AK, KN. c) Gọi I là giao điểm của AM và CB. Tính độ dài IM. Bài 59. Cho đường tròn tâm O bán kính 6cm. Một điểm A nằm bên ngoài đường tròn sao cho các tiếp tuyến AB, AC với đường tròn vuông góc với nhau (B, C là các tiếp điểm). Trên hai cạnh AB, AC lấy các điểm D, E sao cho AD = 4cm, AE = 3cm. Chứng minh rằng DE là tiếp tuyến của đương tròn (O). Ôn tập Bài 60. Cho đường tròn (O) đường kính AB và tia tiếp tuyến Ax. Từ M trên Ax kẻ tiếp tuyến thứ hai MC với đường tròn. Đường vuông góc với AB tại O cắt BC ở N. a) Có nhận xét gì về tứ giác OMNB? b) Tìm tập hợp điểm H là trực tâm tam giác MAC khi M di động trên Ax. HD a) H.b.b do MO song song và bằng BN
  • 22. Hình học kì 1 nâng cao Thầy Hồng Trí Quang 22 b) Tứ giác AOCH là b.b.h nên AH = OC = R. Quỹ tích là (A; R). Bài 61. Đề thi chọn hsg toán 9 quận Tân Bình 2003 - 2004 Cho tam giác ABC nội tiếp đường tròn tâm O đường kính AC. Trên tia AB lấy điểm D sao cho AD = 3AB. Đường thẳng Dy vuông góc với DC tại D cắt tiếp tuyến Ax của đường tròn (O) tại E. Chứng minh tam giác BDE là tam giác cân. Cách 1. Gọi F là giao điểm CD với (O), H là giao điểm AF và BC, I là trung điểm BD Chứng minh AHDE là hình bình hành, suy ra EIKB cũng là h.b.b, từ đó EI vuông BD Cách 2. Hạ EI vuông BD, vẽ JM vuông AD thì M là trung điểm AD
  • 23. Hình học kì 1 nâng cao Thầy Hồng Trí Quang 23 Cách 3. Hạ EI vuông góc BD, tam giác IDE đồng dạng BCD (g.g). Suy ra ID IE BC BD  Tương tự IA IE BC AB  Vậy 1 2 ID ID BC IE AB IA BC IA BD IE    Bài 62. Cho đường tròn (O) đường kính AB và tia tiếp tuyến Ax. Từ M thuộc Ax kẻ tiếp tuyến thứ hai MC với đường tròn. Đường vuông góc với AB tại O cắt BC ở N. a) Có nhận xét gì về tứ giác OMNB b) Tìm tập hợp trực tâm H của tam giác MAC khi M di động trên Ax HD a) h.b.h. b) Cm AOCH là h.b.h mà có OA = OC nên H ở nửa đường tròn tâm A bán kính R nằm trong nửa mp bờ AB chứa Ax mà trừ điểm O. Tự luyện Bài 63. Cho hình thang ABCD vuông tại A và D, tia phân giác của góc C đi qua trung điểm I của AD. a) Chứng minh rằng BC là tiếp tuyến của đường tròn (I; IA) b) Cho AD = 2a. Tính tích của AB và CD theo a c) Gọi H là tiếp điểm của BC và đường tròn (I) nói trên. K là giao điểm của AC và BD. Chứng minh rằng HK song song với DC
  • 24. Hình học kì 1 nâng cao Thầy Hồng Trí Quang 24 Hd a) Kẻ IH vuông BC, chứng minh IH = R b) Chứng minh góc BIC vuong có AB.CD = HB.HC = IH2 c) Theo Talet BH BA BK HC DC KC   Bài 64. Cho tam giác ABC cân tại A, gọi O là trung điểm BC. Vẽ đường tròn tâm O tiếp xúc với AB, AC tại H, K. Một tiếp tuyến đường tròn (O) cắt các cạnh AB, AC ở M và N. a) Cho B C  . Tính MON ? b) Chứng minh rằng OM, ON chia tứ giác BMNC thanh ba tam giác đồng dạng. c) Cho BC = 2a, tính tích BM.CN d) Tiếp tuyến MN ở vị trí nào thì tổng BM + CN nhỏ nhất? HD tích không đổi nên tổng nhỏ nhất khi Bài 65. Cho tam giác ABC nội tiếp (O), có góc C bằng 450 . Đường tròn đường kính AB cắt các cạnh AC, BC lần lượt tại M và N. a) Chứng minh rằng MN OC b) Tính AB MN HD tam giác MBC cân tại M nên MB = MC, mà OB = OC nên MO vuông góc với BC. Tương tự suy ra O là trực tâm tam giác MBC. Đpcm b) tứ giác MONB là hình thang cân nên OB = MN. Góc AOB = 2 góc C = 900 . 2 AB MN  Bài 66. *Cho đoạn thẳng AB. Trên cùng một nửa mặt phẳng bờ AB vẽ nửa đường tròn (O) đường kính AB và các tiếp tuyến Ax, By. Qua điểm M thuộc nửa đường tròn này, kẻ tiếp tuyến cắt Ax, By thứ tự tại C và D. Gọi N là giao điểm của AD và BC. Chứng minh rằng MN vuông góc với AB Hd: Cm MN song song AC theo đảo định lí Talet Bài 67. *Cho đường tròn tâm O, điểm K nằm bên ngoài đường tròn. Kẻ các tiếp tuyến KA, KB với đường tròn (A, B là các tiếp điểm). Kẻ đường kính AOC. Tiếp tuyến đường tròn (O) tại C cắt AB tại E. Chứng minh rằng: a) Các tam giác KBC và OBE đồng dạng b) CK vuông góc với OE
  • 25. Hình học kì 1 nâng cao Thầy Hồng Trí Quang 25 Hd a) BE OB tan BEC tanOKB BC KB    ; b) Gọi I là giao điểm BC và OE, H là giao CK và OE. Xét hai tam giác IBE và IHC ta có góc H vuông, đpcm ĐƯỜNG TRÒN NỘI TIẾP TAM GIÁC Đường tròn tiếp xúc với ba cạnh của tam giác gọi là đường tròn nội tiếp tam giác, còn tam giác gọi là ngoại tiếp đường tròn. Tâm của đường tròn nội tiếp tam giâc là giao điểm ba đường phân giác các góc trong của tam giác. Tính chất 1: Cho tam giác ABC. Gọi D, E, F thưo thứ tự là các tiếp điểm của đường tròn nội tiếp (O) với AB, AC, BC. Chứng minh rằng AB AC BC AD AE 2     Bài 68. Cho tam giác ABC (AC > AB) và trung tuyến AD. Các đường tròn nội tiếp tam giác ABD và tam giác ADC tiếp xúc với AD tại E và K tương ứng. Chứng minh rằng: 𝐴𝐶 − 𝐴𝐵 = 2𝐸𝐾. Bài 69. Cho tam giác ABC có ba cạnh tương ứng là a, b, c. Đường tròn (O) nội tiếp tam giác và tiếp xúc cạnh AB tại D. Tính số đo góc C biết: AC.BC = 2AD.DB Đs vuông tại C Trường hợp đặc biệt, bán kính trong tam giác vuông Bài 70. Cho tam giác ABC vuông tại A Gọi S là diện tích tam giác, R là bán kính đường tròn ngoại tiếp. Chứng minh R r 2S  Từ đẳng thức 2r = b + c – 2R suy Đpcm Tính chất 2: Ứng dụng diện tích Bài 71. Cho tam giác ABC vuông tại A, AB = 5. Tính độ dài AC, BC biết rằng số đo chu vi bằng số đo diện tích. Từ gt r (a b c) a b c 2      nên r = 2. Từ đó b + c – a = 2.r nên a = 13, b = 12. Bài 72. Gọi a b ch ;h ;h là các đường cao tương ứng của một tam giác; r là bán kính đường tròn nội tiếp. Chứng minh các bất đẳng thức sau a) a b ch h h 9r   b) 2 2 2 a b ch h h 27r   Ta có a b c 1 1 1 1 1 1 h h h 2S r(a b c) 9r a b c a b c                      
  • 26. Hình học kì 1 nâng cao Thầy Hồng Trí Quang 26 2 2 2 a b c a b ch h h 3(h h h ) 27r      Tính chất 3: Tỉ số phân giác và góc ở tâm 0A BIC 90 2   Bài 73. Cho tam giác ABC vuông tại A Nếu AB = 9cm, AC = 12cm. Gọi I là tâm của đường tròn nội tiếp, G là trọng tâm tam giác. Tính IG HD Sử dụng định lí đảo talet, gọi BI cắt AC tại N thì BI 2 BG BN 3 BM   nên IG//AC. Vậy IG = 1cm Bài 74. Cho tam giác ABC vuông tại A (không cân). Gọi E là trung điểm của BC, I là tâm đường tròn nội tiếp tam giác ABC, tam giác IEC vuông. Tính tỉ số giữa các cạnh của tam giác ABC. Hd: Chứng minh tam giác IEC vuông tại I; Gọi D là giao CI và AB; có góc BID bằng 45 suy ra BIE = 45. Suy ra tam giác BDI = BEI suy ra BC = 2 BE = 2BD Áp dụng tính chất phân giác, ta có AC = 2AD; Từ đó BC + AC = 2AB. Kết hợp với Pitago ta có 3 4 5 AC AB BC   Tự luyện Bài 75. Cơ bản Cho tam giác ABC cân tại C, nội tiếp đường tròn tâm (O) bán kính R = 1. Cạnh bên AC gấp hai lần cạnh đáy BC. Đường tròn tâm I nội tiếp trong tam giác. Tính bán kính của đường tròn (I). HD Sử dụng Pitago tính 15 4 CM  ; Ta có 5CD CI ID r   ; 15 4 1; 8 OD r AD   . Vậy 3 8 r 
  • 27. Hình học kì 1 nâng cao Thầy Hồng Trí Quang 27 Bài 76. Từ một điểm A ở bên ngoài đường tròn (O) ta vẽ hai tiếp tuyến AB, AC với đường tròn (B, C là các tiếp điểm). Trên AO lấy điểm M sao cho AM = AB. Các tia BM và CM lần lượt cắt đường tròn tại một điểm thứ hai là D và E. Chứng minh rằng: 1/ M là tâm đường tròn nội tiếp tam giác OBC 2/ DE là đường kính của đường tròn (O). HD 1/ Chứng minh 𝐶𝐵𝑀̂ = 𝑂𝐵𝑀̂ cùng phụ với hai góc bằng nhau 2/ OE // BC (2 góc sole trong bằng nhau); tương tự OD // BC. Vậy E, O, D thẳng hàng. Bài 77. Tam giác ABC vuông tại A ngoại tiếp đường tròn (I; r). Gọi G là trọng tâm tam giác. Tính các cạnh của tam giác ABC theo r biết IG song song với AC Bài 78. Cho tam giác ABC vuông tại A có đường cao AH. Gọi (O; r), (𝑂1; 𝑟1), (𝑂2; 𝑟2) theo thứ tự là các đường tròn nội tiếp các tam giác ABC, ABH, ACH. a) Chứng minh rằng: 𝑟 + 𝑟1 + 𝑟2 = 𝐴𝐻 b) Chứng minh rằng 𝑟2 = 𝑟1 2 + 𝑟2 2 c) Tính 𝑂1 𝑂2 nếu biết AB = 3, AC = 4 HD a) Tính 𝑟 + 𝑟1 + 𝑟2 theo ba cạnh, b) Sử dụng tỉ số đồng dạng và tỉ lệ thức, c) Áp dụng pitago có    1 2 1 2 2 22 1 2 r rOO r r  1 2 2OO  Bài 79. Cho tam giác ABC có chu vi 80cm ngoại tiếp đường tròn (O). Tiếp tuyến của đường tròn (O) song song với BC cắt AB, AC theo thứ tự tại M và N. a) Cho biết MN = 9,6 cm. Tính độ dài BC b) Cho biết AC – AB = 6cm, tính độ dài các đoạn AB, AC, BC để MN đạt giá trị lớn nhất. HD AMN ABC PMN BC P  Đặt BC = x, suy ra BC = 24 hoặc BC = 16cm b) 40.MN = x (40 – x) 2 (x 20) 400 400     nên max MN = 10 ; x = 20 Từ đó BC = 20, AB = 27, AC = 33. Công thức: 𝑆 = 𝑃𝑟 Bài 80. *Cho tam giác ABC ngoại tiếp đường tròn (O). Gọi D, E, F theo thứ tự là tiếp điểm trên cách cạnh BC, AB, AC. Gọi H là chân đường vuông góc kẻ từ D đến EF. Chứng minh rằng: BHE CHF Hd Kẻ BI, CK vuông với EF, chứng minh BEI đồng dạng CFK, tiếp đó BHI đồng dạng CHK
  • 28. Hình học kì 1 nâng cao Thầy Hồng Trí Quang 28 Bài 81. *Cho tam giác ABC. Đường tròn (O) nội tiếp tam giác tiếp xúc với BC tại D. Vẽ đường kính DN của đường tròn (O). Tiếp tuyến của đường tròn (O) tại N cắt AB, AC theo thứ tự tại I, K. a) Chứng minh rằng: NI DC NK DB  b) Gọi F là giao điểm của AN và BC. Chứng minh rằng BD = CF. Hd a) tam giác NOI đồng dạng DBO nên 2 NI.DB r NK.DC  b) Ta có NK NI NK NI IK AK NF DB DC DB DC BC AC FC        từ đó FC = DB Bài 82. *Cho đường tròn (O) nội tiếp tam giác đều ABC. Một tiếp tuyến của đường tròn cắt cạnh AB và AC theo thứ tự tại M và N. a) Tính diện tích tam giác AMN biết BC = 8, MN = 3 b) Chứng minh rằng: 𝑀𝑁2 = 𝐴𝑀2 + 𝐴𝑁2 − 𝐴𝑀. 𝐴𝑁 c) *Chứng minh rằng: 1 AM AN MB NC   Bài 83. *Cho tam giác ABC có BC = a, AC = b, AB = c. Gọi (I) là đường tròn nội tiếp tam giác. Đường vuông góc với CI tại I cắt AC, AB theo thứ tự tại M và N. Chứng minh rằng: a) 2 2 AM.BN IM IN  b) 2 2 2 IA IB IC 1 bc ca ab    Hd a) Tam giác AMI đồng dạng AIB, AIB đồng dạng INB nên AMI đồng dạng INB nên AM.BN = IM . IN
  • 29. Hình học kì 1 nâng cao Thầy Hồng Trí Quang 29 b) Đặt AM = m, BN = n, IM = IN = x. Do tam giác AMI và AIB đồng dạng nên AM.AB = IA2 =mc 2 IA m bc b  . Tương tự 2 IB n ca a  Theo Pitago 2 2 2 IC CM IM  (b m)(a n) mn    ab bn am   Do đó 2 2 2 IC n m IA IB 1 1 ab a b bc ca       ĐƯỜNG TRÒN BÀNG TIẾP Bài 84. Đtròn bàng tiếp *Gọi D, E, F là tâm các đường tròn bàng tiếp của tam giác ABC. Chứng minh tam giác DEF là tam giác nhọn. HD Gọi I là tâm nội tiếp, suy ra CE vuông CD; IA vuông IE nên AEC bằng AID mà AID bằng nửa tổng A + C nhọn nên DEF nhọn VỊ TRÍ TƯƠNG ĐỐI CỦA HAI ĐƯỜNG TRÒN Hai đường tròn cắt nhau Tính chất: Trung trực Bài 85. Cho ba đường tròn 1 2 3; ;O O O có cùng bán kính R và cùng đi qua một điểm I. Gọi các giao điểm khác I của ba đường tròn      1 2 2 3 3 11 ; ; ; ; ;O O O O O O tương ứng là A, B, C. Chứng minh rằng: a) 1 2 3ABC OO O   b) Đường tròn ngoại tiếp tam giác ABC cũng có bán kính R. c) I là trực tâm tam giác ABC. d) Các đường 3 2 1; ;AO BO CO đồng quy. Bài 86. Cho hai đường tròn (O) và (O’) có cùng bán kính, cắt nhau tại A và B. Kẻ cát tuyến chung DAE của hai đường tròn (D thuộc (O), E thuộc (O’)). Chứng minh rằng BD = BE. Nxet: Bài này hơi khó, nếu học kì 2 sử dụng tứ giác nội tiếp sẽ đơn giản hơn.
  • 30. Hình học kì 1 nâng cao Thầy Hồng Trí Quang 30 Gọi I là giao điểm AB và OO’, hạ OH, O’K, IM vuông góc với DE. Chứng minh tam giác HIK cân do IM là trung tuyến đồng thời là đường cao. Do đó BDE cân. Hai đường tròn tiếp xúc Tính chất OO’ = R + R’ Bài 87. Cho hai đường tròn (O; R) và (O’; R’) tiếp xúc ngoài nhau tại M (R > R’). AB là tiếp tuyến chung ngoài (A thuộc (O), B thuộc (O’)). a) Tính độ dài AB theo R, R’ Đs 2 'AB RR b) Gọi C là giao điểm của MB với (O). Chứng minh ba điểm A, O, C thẳng hàng c) Tính độ dài MA, MB. d) Nếu điểm M cố định, bán kính R, R’ không đổi. Hai điểm O, O’ thay đổi sao cho hai đường tròn (O; R) và (O’; R’) tiếp xúc nhau tại M. Tìm quỹ tích trung điểm I của BC. HD cách 1. Gọi M là điểm tiếp xúc, tiếp tuyến tại M cắt AB tại I thì AB = 2IM và 2 . 'IM OM O M Cách 2. Từ O’ hạ O’H vuông góc với OA, sử dụng định lí Pitago. Bài 88. Áp dụng bài trên Cho hai đường tròn (O;R) và (O’;R’) tiếp xúc ngoài tại A. Gọi MN là tiếp tuyến chung ngoài. Đường tròn (I;r) tiếp xúc với MN và tiếp xúc ngoài với hai đường tròn (O) và (O’). Chứng minh rằng 1 1 1 'r R R   𝐻𝐷 𝑎) 𝑂′ 𝑂2 = 𝑂′ 𝐾2 + 𝑂𝐾2 ; 𝑏) 𝑀𝑁 = 𝑀𝐸 + 𝐸𝑁 Hai đường tròn không cắt nhau
  • 31. Hình học kì 1 nâng cao Thầy Hồng Trí Quang 31 Bài 89. Cho hai đường tròn ngoài nhau (O) và (O’). Kẻ tiếp tuyến chung ngoài MM’và tiếp tuyến chung trong NN’ của hai đường tròn (M, N là tiếp điểm của đường tròn (O); M’, N’ là tiếp điểm của đường tròn (O’)). Gọi giao điểm của MM’ và NN’ là P, giao điểm của MN và M’N’ là Q. a) Tính góc OPO’ b) Gọi AB là tiếp tuyến chung ngoài thứ hai, NN’ cắt AB tại K. Chứng minh KN = PN’. c) Chứng minh ba điểm O, Q, O’ thẳng hàng. HD a) Phân giác của hai góc kề bù b) Góc MQM’ vuông, chứng minh tổng hai góc còn lại bằng 900 . Sai lầm khi sử dụng đồng vị, cho góc Q bằng góc O’. ' OQ OO     Tự luyện Hai đường tròn cắt nhau Bài 90. Cho điểm A nằm ngoài đường tròn (O). Vẽ đường tròn tâm A bán kính AO. Gọi CD là tiếp tuyến chung của hai đường tròn, C thuộc (O), D thuộc (A). Đường nối tâm OA cắt đường tròn (O) ở H. Chứng minh rằng DH là tiếp tuyến của (O). Tiếp xúc Bài 91. Cho hai đường tròn (O) và (O’) tiếp xúc ngoài tại A. Gọi AB là đường kính của đường tròn (O); AC là đường kính của đường tròn (O’); DE là tiếp tuyến chung của hai đường tròn, D thuộc (O), E thuộc (O’); K là giao điểm của BD và CD. a) Tứ giác ADKE là hình gì? b) Chứng minh AK là tiếp tuyến chung của hai đường tròn (O) và (O’).
  • 32. Hình học kì 1 nâng cao Thầy Hồng Trí Quang 32 c) Gọi M là trung điểm BC. Chứng minh rằng MK vuông góc với DE. Bài 92. Hai đường tròn (O;R) và (O’;R’) tiếp xúc ngoài tại A. Gọi BC, DE là các tiếp tuyến chung của hai đường tròn (B, D thuộc đường tròn tâm (O)). a) Chứng minh BDEC là hình thang cân b) Tính diện tích hình thang cân đó. Bài 93. Cho nửa đường tròn tâm O đường kính AB. Gọi OC là bán kính vuông góc với AB, d là tiếp tuyến với nửa đường tròn tại C. Gọi (I) là đường tròn tiếp xúc trong với nửa đường tròn tâm O và tiếp xúc với đường kính AB. Chứng minh rằng điểm I cách đều đường thẳng d và điểm O. Hai đường tròn không giao nhau Bài 94. Cho hai đường tròn (O) và (O’) ở ngoài nhau. Qua (O) kẻ các tia tiếp tuyến với đường tròn (O’), chúng cắt (O) tại A, B. Qua (O’) kẻ các tiếp tuyến với (O) chung cắt (O’) tại C và D. Chứng minh rằng A, B, C, D là bốn đỉnh của hình chữ nhật. MỘT SỐ BÀI TOÁN VẼ THÊM ĐƯỜNG PHỤ Vẽ đường kính vuông góc với một dây cung Bài 95. Cho góc xOy có số đo bằng 600 . Lấy điểm E trên tia phân giác của góc đó. Vẽ đường tròn (E) cắt tia Ox tại A và B, cắt tia Oy tại C và D sao cho OA < OB, OC < OD. Vẽ đường tròn (F) đi qua ba điểm E, A, D. Chứng minh rằng: 1/ AB = CD 2/ 𝐴𝐸𝐷̂ = 1200 3/ F thuộc đường tròn (E) HD 1/ Từ E hạ EH, EK vuông góc AB, CD, ta có EH = EK; 3/ Chứng minh tam giác AEF cân tại F có góc ở đáy 600 nên đều. từ đó EA = EF. Vẽ tiếp tuyến chung tại điểm tiếp xúc của hai đường tròn Bài 96. Cho nửa đường tròn (O) đường kính AB. Vẽ bán kính OC vuông góc với AB (C thuộc đường tròn). Từ C vẽ tiếp tuyến xy với nửa đường tròn. Vẽ đường tròn (K) tiếp xúc với AB và tiếp xúc với (O). Chứng minh rằng điểm K luôn cách đều điểm O và đường thẳng xy. HD Gọi M, N lần lượt là tiếp điểm của (K) với AB và (O). Từ N vẽ tiếp tuyến chung của hai đường tròn cắt AB tại D và cắt MK tại E. Ta chứng minh E thuộc xy DEM DON   nên EM = ON = OC suy ra tứ giác ECOM là h.b.h tức E thuộc xy Từ EM = ON nên KE = KO, đpcm
  • 33. Hình học kì 1 nâng cao Thầy Hồng Trí Quang 33 Bài 97. **Cho hai đường tròn (O) và (O’) cắt nhau tại A, B. Qua A kẻ hai cát tuyến cắt (O) tại C và P, cắt (O’) tại D và Q tương ứng. Chứng minh rằng CD = PQ khi và chỉ khi AB là phân giác của góc 𝑃𝐴𝐷̂ Luyện tập Bài 98. Cho đường tròn (O; R) dây AB bất kì và tiếp tuyến Ax. Vẽ BH vuông góc với Ax. Chứng minh rằng tỉ số 2 AB BH không đổi HD Kẻ OK vuông AB thì 2 AB 2OA.BH Bài 99. Cho tam giác ABC vuông cân tại A, BC 2 2cm . Vẽ các đường tròn (B; BA) và (C; CA). Một đường thẳng bất kì đi qua A cắt đường tròn (B) và (C) lần lượt tại D và E. Chứng minh rằng: 2 2 AD AE không đổi HD Kẻ BH, CK vuông DE thì BH = AK nên 2 2 AD AE 2 2 2AD AE 4 4 4.AB 16 2 2                Bài 100. Cho hai đường tròn đồng tâm (O) có bán kính R và r (R > r). Gọi A là một điểm cố định trên đường tròn nhỏ. Qua A vẽ đường thẳng xy cắt đường tròn lớn tại B và C, cắt đường tròn nhỏ tại một điểm thứ hai là D 1/ Chứng minh rằng AB = CD 2/ Vẽ dây AM của đường tròn nhỏ sao cho AM vuông góc xy. Chứng minh rằng trọng tâm G của tam giác MBC là một điểm cố định 3/ Chứng minh rằng khi xy xoay quanh A thì tổng: 2 2 2 AM AB AC  không đổi. HD 1/ Kẻ OH vuông BC; 2/ G chia OA theo tỉ lệ 1:2 nên cố định 3/ 2 2 2 AM AB AC  2 2 2 (2OH) (HB AH) (HC HA)     2 2 2 2 2(OH HB ) 2(OH HA )    2 2 2(R r )  Bài 101. *Cho đường tròn (O; 1). Lấy một điểm A cố định trên đường tròn. Vẽ tam giác MAB vuông tại M, AB là một dây cung của đường tròn (O). Tìm giá trị lớn nhất của độ dài OM HD Từ O hạ OH vuông AB thì HA = HB = HM = x. Xét ba điểm H, O, M ta có: OM OH HM  2 1 x x       2 2 2 2 2 1 1 1 x x 2           Dấu bằng xảy ra khi AB 2 và H, M, O thẳng hàng (tức tam giác AOB vuông tại O)
  • 34. Hình học kì 1 nâng cao Thầy Hồng Trí Quang 34 Bài 102. Cho tam giác ABC vuông tại A, đường tròn (O) nội tiếp có bán kính r. Vẽ đường thẳng d đi qua O cắt hai cạnh AB, AC lần lượt tại M và N. Xác định vị trí của d để tam giác AMN có diện tích nhỏ nhất? HD gọi S là diện tích tam giác AMN. Ta có: 2S = AM.AN AOM AON 1 S S S r(AM AN) 2     1 .r.2. AM.AN 2  r 2S S r 2S 2 S 2r  Dấu bằng xảy ra khi và chỉ khi AM = AN tức d vuông góc OA Bài 103. Cho đường tròn (O; R). Từ một điểm A cách O là 2R vẽ hai tiếp tuyến AD, AE với đường tròn (D, E là tiếp điểm). Trên cung nhỏ DE lấy điểm F. Qua F vẽ tiếp tuyến thứ ba với đường tròn cắt các tia AD, AE lần lượt tại M, N. Qua O vẽ một đường thẳng vuông góc với OA cắt các tia AD, AE lần lượt tại B và C. 1/ Chứng minh rằng ABC ACB MON  2/ Chứng minh rằng: 2 BM.CN OB 3/ Xác định vị trí của điểm F để tổng BM + CN có giá trị nhỏ nhất? HD Gọi 2 tiếp điểm là D và E; ta có: 1 180 A MON DOE B C 2 2      2/ Tam giác MBO MON MBO OCN     2 BM.CN OB  3/ 2R 3 OB 3  ; 4R 3 BM CN 2 BM.CN 3    Dấu bằng xảy ra khi F là giao điểm của AO với đường tròn. Bài 104. Cho các đường tròn (O; R) và (O; R’) có bán kính khác nhau, tiếp xúc ngoài nhau tại M. Vẽ các tiếp tuyến chung ngoài AB, CD với hai đường tròn trong đó A, D thuộc đường tròn (O); B, C thuộc đường tròn (O’). Chứng minh rằng: 1/ Ba đường AB, CD, OO’ đồng quy 2/ AB + CD = BC + AD. HD 1/ Gọi giao AB và CD cắt nhau tại K, thì O, O’ đều nằm trên đường phân giác AKB
  • 35. Hình học kì 1 nâng cao Thầy Hồng Trí Quang 35 2/ Từ M kẻ tiếp tuyến chung của 2 đường tròn cắt AB, CD tại E và T. Theo t/c tiếp tuyến ta có: EA = EM = EF; FM = FD = FC. Theo t/c đường trung bình hình thang: AD BC 2EF AB DC    Bài 105. Cho đoạn thẳng AB = 2a. Đường tròn (C) tiếp xúc với AB tại A, đường tròn (D) tiếp xúc với AB tại B, hai đường tròn này thay đổi nhưng luôn thuộc cùng một nửa mặt phẳng bờ AB và luôn tiếp xúc ngoài với nhau tại M. 1/ Hỏi điểm M di động trên đường nào? 2/ Chứng minh rằng: 2 AC.BD a HD M nằm trên đường tròn (O; a); 2/ Ta có: 2 2 AC.BD CM.DM OM a   Bài 106. *Cho đường tròn (O) và một điểm P trong đường tròn. Qua P kẻ cát tuyến APB bất kì. Đường tròn tâm I qua A, P tiếp xúc với (O) tại A, đường tròn tâm J qua B và P tiếp xúc với (O) tại B. Giao điểm thứ hai của hai đường tròn (I) và (J) là M. Chứng minh rằng đường thẳng IJ luôn đi qua một điểm cố định khi cát tuyến APB thay đổi. Từ đó suy ra tập hợp các điểm M nằm trên một đường tròn cố định. Hd góc API = ABJ suy ra PI song song BO Tương tự suy ra PIOJ là h.b.h hay IJ đi qua K là trung điểm OP Vì PM vuông góc với JI nên M thuộc đường tròn đường kính OP